You are on page 1of 53

MEDICINE

1. In dislocation of shoulder, which nerve is liable to be injured?


a. Radial
b. Axillary
c. Musculocutaneous
d. Ulnar
e. Brachial plexus

2. Which of the following is likely to damage an artery?


a. Anterior Dislocation shoulder
b. Posterior dislocation shoulder (Neurovascular deficits are infrequent)
c. Anterior dislocation hip
d. Posterior dislocation hip (The sciatic nerve and the common peroneal division of the sciatic nerve are most often injured in
posterior dislocations)
e. Posterior dislocation knee (Knee dislocation is a relatively rare injury but an important one to recognize because coexistent
vascular injury, if missed, often leads to limb loss. A careful vascular examination is
essential, as popliteal artery injury occurs in 7-45% of all knee dislocations)

3. In Colles Fracture, the distal segment is displaced:

a. Backwards and ulnarwards


b. Backwards and radially
c. Forwards and ulnarwards
d. Forwards and radially
e. Minimally displaced

4. An 18-month-old is brought to you by the mother for breast enlargement. Tanner 3. No pubic hair or
axillary hair is seen. No other abnormality is seen. What is your diagnosis? (Tanner III= breast begins to become
more elevated, and extends beyond the borders of the areola, which continues to widen but remains in contour with surrounding breast
. (Puberty normally occurs between 13 and 15 years old in boys, and between 9 and 16 years old in girls.)
a. Precocious puberty (usually associated with pubic and axillary hair and developed genitalia)
b. Idiopathic thelarche (Thelarche is the first stage of secondary (postnatal) breast development, usually occurring at the
beginning of puberty in girls.)(In Premature Thelarche early enlargement of the breast or breasts occurs between ages 1 and 3 years. In
some girls, enlarged breasts are present at birth, there are no other signs of early puberty, except the breast development.)
c. Congenital adrenal hyperplasia (Newborn girls with this disorder have a swollen clitoris with the urethral opening at the
base. As the female grows older, some features start to appear male,)
d. Breast tumour
e. Mastitis

5. A 2-year-old child is brought to you for having loose motions for 3 days. The stool contains undigested
food material. No vomiting. Thriving well. What is he suffering from?
a. Cystic fibrosis (Failure to thrive, meconium ileus, rectal prolapse)
b. Coeliac disease (steatorrhea and offensive stools with loss of weight and iron deficinecy anemia)
c. Gastroenteritis
d. Toddlers diarrhoea (is the most common cause of chronic diarrhoea in young children between the ages of 1 and 5)
e. IBS (undigested food material is a charecteristic of IBS)

6. A large man presents with big face broad hands thick and oily skin and hoarse voice. What is the initial
investigation of choice?
a. GTT + GH (Oral Glucose Tolerancs test + GH)
b. Insulin challenge test + GH
c. GH+ cortisol
d. GH
e. GTT

7. Regarding heparin induced thrombocytopenia, what is true? (thrombocytopaenia and associated thrombotic events
are relatively common side effects of heparin therapy.)
a. Purpura
b. Joint bleeding
c. Ecchymosis
d. Thrombosis
e. Hematuria

8. Regarding ADHD, what is correct? (Attention Deficit Disorder with Hyperactivity)

a. Age 5-7 years


b. Reading difficulties
c. Social withdrawal
d. Females affected more than Males
e. Prevalence is 15%

9. All the following can cause dysphagia except:


(AMCQ ME-Q65) a.
Oesophageal cancer
b. Achalasia
c. Oesophageal varices
d. Barrett’s oesophagus
e. Hiatus hernia

10.A 6 week old is brought to you for not having passed motion for last 4 days. He has been feeding well,
and is gaining weight. Otherwise well and active. What is the most probable explanation?

a. Hirsprung’s disease (Hirschsprung's disease is characterised by failure of passage of meconium during the first 24 hours)
b. Normal variant
c. Hypothyroidism
d. Acquired constipation
e. Foreign body

11. A man returns after a trip to India. Now has 3 days diarrhoea with blood in stools. Looks run out and
dehydrated. Noted to have fever. What is most correct? (Causes of infectious bloody diarrhea: Salmonella, Shigella,
Campylobacter jejuni, Yersinia enterocolitica, E. coli and Entamoeba histolytica)
a. Giardia infection
b. Amoebiasis
c. E. coli gastroenteritis (AMCQ ME Q75) (usually lasts 48-72 hours)
d. Viral diarrhoea
e. Cholera (watery diarrhea "rice water" appearance -- the stool looks like water with flecks of rice in it)

12. In a 40 years old lady, all the following may be the cause of menorrhagia except:
a. Hormonal response to uterus
b. Subserous fibroid (menorrhagia - associated with intramural and submucous fibroids)
c. Intermural fibroid
d. Adenomyosis
e. Endometriosis
13. 3 month old comes with unilateral eye discharge. What is the most possible cause:

a. Chlamydia infection
b. Gonococcus
c. Ophthalmia neonatorum
d. Blocked nasolacrimal duct
e. Normal variant

*14. A 6-month-old presents with SOB. There was a birthday party at home yesterday. On examination,
child is wheezing. What is most initial investigation?
a. Sweat test
b. Bronchogram
c. CXR
d. Inspiratory and Expiratory Rontogram
e. PEFR measurement

15. A mother brings her 3-year-old baby girl with unilateral nasal discharge off and on for last 3 months.
Secretion is foul smelling and at times copious. Baby is uncooperative. What is the initial investigation of
choice?
a. CXR
b. indirect laryngoscopy
c. Examination under GA

16. A 6-year-old girl wakes up at night c/o sever pain in her calves. Mother rubs her calves and she feel
fine, goes to sleep. This has been happening for a number of days now. At daytime she is active and
playing. All of the following are true except:
a. Ca and CK will be abnormal
b. No investigations are needed
c. Family Hx will be positive
d. Reassurance is the treatment.

17. Which vaccination should not be given to a child who is under remission on chemotherapy for CLL?
(Live, attenuated vaccines include: measles, mumps and rubella, BCG, poliomyelitis - oral Sabin vaccine, yellow fever, typhoid –
oral)
a. Polio
b. MMR
c. HiB
d. DPT

18.Solitary mobile carcinoma breast lump. No axillary lymphadenopathy. Which of the following is a
prognostic marker?
a. Age > 45
b. Neu2 / HER oncogene status
c. Oestrogen level

19. All can occur with chronic limb ischemia except:


a. Great Toe gangrene
b. Pallor
c. Ulcer on medial side of leg above malleolus (venous ulcer)
d. Rest pain
e. Intermittent claudication

*22. 40 year old with acute painful testis:*


a. Varicocele
b. Hydrocele
c. Epididymoorchitis
d. Torsion (usually occur in young ages)
e. Tumour(painless)

23. WOF is true regarding a 2cm kidney stone in the pelvis seen in KUB examination.:
a. Expectant treatment is right (in stones less than 5mm)
b. It is a urate stone
c. Lithotripsy (Indicated in stones ranging from 2-4 providing that the collecting system is normal)
d. Open surgery

24. Patient can’t dorsiflex or evert his foot. What is wrong?


(foot drop is due to weakness of tibialis ant muscle which is supplied by the L5 root through the common peroneal nerve)
a. L4
b. L5
c. Tibial nerve (S1+S2)
d. Peroneal nerve (L4+L5+S1)
e. Sciatic nerve (L4+L5+S1+S2)

25. Body dysmorphia is seen in AOF except: (Normal perception of body image is disturbed by localized lesions of the
posterior nondominant parietal lobe as well as the temporal lobe)
a. Anorexia nervosa
b. Acromegaly
c. Non dominant parietal lobe lesion
d. Dominant parietal lobe lesion
e. Narcissistic personality disorder

26. 6 year old child comes to you with up rolling if eyes, neck stiffness, protruded tongue, rigid body and
face tilted to one side. Had vomiting and was being treated by the GP. What is the cause?
a. Huntington’s chorea
b. Epilepsy
c. Reaction to metoclopramide
d. Gastritis

27. Lung cancer associated with non-smokers:


a. Sq. cell Ca.
b. Adenocarcinoma
c. Small cell Ca.
d. Oat cell Ca.
e. Large cell Ca.

28. All of the following cause gynecomastia except :

a. spironolactone
b. cimetidine
c. digoxin
d. alpha methyldopa (Hyperprolactinemia (gynecomastia, galactorrhea)
e. labetolol

29.A young female presents with 24 hour h/o sore throat. Later she develops proteinuria and some
hematuria. What is the most probable cause?
a. Glomerulonephritis (may develop 1-2 weeks after an untreated throat infection, or 3-4 weeks after a skin infection.)
b. IgA Nephropathy (Recurrent Bloody urine may begin during or soon after a respiratory infection)
c. Nephrotic syndrome (Characterised by protinuria, albuminaemia and edema NO hematuria)
d. HTN

30.What is the most important step in psychotherapy?

a. Detailed history
b. Isolation
c. History from friends and relatives.
d. Developing relationship
e. Investigations

31. When do we use Psychodynamic Psychotherapy?


a. Obsessive Compulsive Disorder
b. Bipolar Disorder
c. Schizophrenia
d. Depression
e. Anorexia Nervosa

32. You are asked to examine a person. He is cut off from the world since he was teen. Lives alone and
wants to be alone. No friends. Family wants to support but he refuses to get any help. He doesn’t have any
thought disorder or perceptional symptoms, but his affect is blunt. What would be the most probable
diagnosis?
a. Chronic schizophrenia
b. Major depression
c. Narcissistic Personality
d. Bipolar disorder

33.Regarding Billing’s method of contraception, AOF are true except:

a. Cervical mucus has to be palpated


b. Unsafe sex, if it becomes more
c. Safe sex after 2-3 days of becoming more
d. Since it considers the cervical mucus, it is same contraception in regular, irregular and lactational
menstrual periods.

34.Regarding Levonorgestral containing IUCD, AOF are true except:

a. Light periods
b. Amenorrhoea
c. Decreased risk of cervical carcinoma
d. Decreased risk of sexually transmitted diseases
e. Decreased risk of ectopic pregnancy

35.Spleenectomy in spherocytosis will do all the following except:


a. Normalize anaemia
b. Normalize spherocytosis

36. An 8-year-old child presents to the ED with bilateral tonsillitis with greyish white exudates. He has
cervical lymphadenopathy, fever and hepatosplenomegaly. What is the probable cause?
a. EBV (infectious mononucleosis: fatigue, malaise, headache, and sore throat with enlarged tonsils covered with a whitish-yellow
covering and enlarged and painful lymph nodes.)
b. CMV
c. Diphtheria
d. Bacterial

37. A young child with 2-week h/o dry cough with whop, and some times vomits. What is true about this
patient? (Pertussis)
a. Will have cough for the next 4 weeks (The infection usually lasts 6 weeks. coughing spells can last for several weeks to
two months or longer)
b. Amoxicillin should be started (treatment is usually started before the results are ready with antibiotics such as
erythromycin and amoxicillin)
c. D & T vaccine should be given immediately

38. A patient with pyloric stenosis, vomiting for 4 days now presents to ED with mother. He is dull
lethargic, skin turgor is lost, and BP is 90/50. What is the most initial choice of fluid in this patient?

a. Normal saline (gastric fluid is similar to body fluids)


b. Hartman’s solution
c. Ringer’s lactate solution
d. Dextrose 5% and Normal saline

39.Regarding diffuse fibrosing alveolitis what is true?

a. Decreased FEV1 / FEV (ratio of FEV1 to vital capacity is not reduced)


b. Decreased TLC
c. Decreased Expansion (there may be slightly reduced chest expansion)
d. Decreased O2 tension
e. Hypercarbia (Defective diffusion affects oxygen but not CO2 where Obstructive pulmonary diseases affect both)

40.Psychiatric side effect of corticosteroids?


Withdrawal causes delirium

41. A patient is suffering from cyclical mastalgia. Routine treatment fails. What is your next management?
(The first line of these is evening primrose oil or gamolenic acid. Second line treatment consists of either danazol or bromocriptine.)
a. NSAID
b. Bromocriptine
c. Danazol
d. Progesterone
e. Lasix

42.Regarding CPR, WOF statement is most correct?


a. Adrenaline should be given every 10 minutes(every 3 min)
b. Bicarbonate is mandatory (only when acidosis is present to avoid hypokalemia)
c. ECM only pumps ½ of the cardiac output in one stroke
(Amount of blood pumped by the normally beating adult heart at rest is 4.5-6.5 liters/minute.)
(Amount of blood pumped by the heart during heart massage is 0-2.2 liters/minute)
d. ECG is necessary before cardio-version. (Cardioversion is not used when the heart is in complete arest but is used
when there is VF/VT, thus the heart’s state should be identified first)

43. Patient with grandiose character. Doesn’t care about others. Feels himself to be unique:

a. Histrionic personality
b. Schizoid
c. Narsistic
d. Delusional
e. Borderline

44.Picture of CT Scan showing two white spots just lateral to the midline. A patient 59 year old presents
with sudden onset of left hand weakness. What could be the possible cause?

a. SDH (Subdural Hematoma: cresent shaped collection of blood at the periphery of the cerebral hemisphere)
b. SAH (subarachnoid hemorrhage appears as linear areas of high attenuation within the cisterns and sulci which differentiates it
from a subdural hematoma which does not extend into the sulci.)
c. Intra cerebral haemorrhage
d. Cerebral infarct (infarcts appear darker than the surrounding tissue)

45. WOF has the least risk of having congenital malformation at birth.
a. Chromosomal abnormality AD, AR, Sex linked recessive (AD:50% chance, AR& sex L:25% chance)
b. Rubella at 18 weeks (Featus is most at risk in the first 16 weeks of gestation. Les than 5% are infected if at or after 16
weeeks.)

46. What is the most likely cause of post coital bleeding?


a. Endometrial Carcinoma (more likely to originate from the vagina or cervix than the endometrium.)
b. Vaginal candidiasis
c. Cervical polyp
d. Fibroid

47. Regarding thyroid cancer, all are true except (The most common presentation of thyroid carcinoma is a painless,
palpable, solitary thyroid nodule. Solitary nodules are more likely to be malignant in patients older than 60 years and in patients
younger than 30 years. Sudden onset of pain has a stronger association with benign disease, such as hemorrhage into a benign cyst or
subacute viral thyroiditis, Hoarseness suggests involvement of the recurrent laryngeal nerve and vocal fold paralysis.)
a. Enlarged thyroid (Characterised by a rapid rate of growth)
b. Solitary nodule
c. Hot nodule
d. Change in voice (suggests involvement of the recurrent laryngeal nerve and vocal fold paralysis.)
e. Tracheal compression

48. Patient presents with fever, tachycardia (Pulse 110/mt), Tremors, and palpable & tender thyroid. On
investigation, there is no radioactive iodine uptake. What is the possible treatment? (thyrotoxicosis)
a. Carbimazole (to block the action of thyroid hormones)
b. I131 (will not be taken by the thyroid gland)
c. Beta-Blocker + Paracetamol (to decrease the heart rate)
d. Thyroxine (will agrevate the condition)

49. A women has bilateral suppurative inflammatory lesion on the axilla and groin. What is true?
a. Actinomycosis (is a chronic infection, commonly of the face and neck, that produces abscesses and open draining sinuses.)
b. Fungal infection - Mycosis Fungoides (occurs more frequently in men than in women. Characterised by Patches and
plaques which affect any area of the skin,specialy thr bathing suit area (ie, hips, buttocks, groin, lower trunk, axillae, breasts)
c. Suppurative Hidradenitis
d. Multiple lymphadenopathy with abscess
e. Pilonidal sinus (is present posterioir to the Anus)

50. An old man who was operated for aortic aneurysm 3 weeks ago, presents to the ED with irregular pulse,
restlessness, abdominal distension and not having passed flatus and motions for last 2 days. WOF is the
most possible cause?
a. Rupture of suture site
b. Mesenteric artery embolism (acute abdominal pain with gross blood in stools and rectum)
c. Urinary retention
d. Diverticulosis

51 Why do we do an ultra sound in a patient with sign symptoms of biliary disease?


a. To visualize stones in the GB or CBD
b. To visualize pancreatic cancer
c. To see the dilation of bile tree
d. To locate the hepatic cancer
e. To visualize the GB pathology

52. A seventy-year lady who is alcoholic and smokes > 20 cigarettes a day, presents to you with tiredness
and fatigue. She also complains of weakness in limbs. Her Hb was 98, MCV 110, and platelets were
decreased. What is the most possible cause? (AMCQ ME-Q90)
a. AoCD
b. IDA
c. Alcoholism (Due decrease Vit B12 and folate absorbtion and metabolism by the liver)
d. Pernicious Anaemia (lack of intrinsic factor)
e. Auto immune Haemolytic Anaemia

53. A young female on oral contraceptive misses one tablet while in the mid cycle. Had coitus the same
day, and takes the ‘missed’ tablet 12 hours later. Then continues with rest of her tablets. She has spotting on
the third day of coitus. What will be your advice?
a. Continue with the OCP.
b. Change to different contraceptive drug.

54. AOF are the features of basal ganglion except:


(Basal gaingilion lesions cuse parkinsonis which is characterised by resting tremor, rigidity, bradykinesia, and postural instability.) a.
Ataxia (cerebellar lesions)
b. Tremor
c. Bradykinesia
d. Chorea
e. Rigidity

55.Most common cause of bowel obstruction in Australia assuming that he as not undergone any abdominal
surgery, is:
a. Volvulus
b. Intersuseption (occure in very young ages)
c. Groin hernia
d. Internal hernia
e. Tumours

56. A patient presents with wasting of small muscles of hand with thenar sparing. What is the most probable
diagnosis?

a. Ulnar N
b. Median N
c. Axillary N
d. Musculocutaneous N

57. Picture on Page 206 of AMC. Picture of scalp showing white scaly lesion.
a. Psoriasis
b. Alopecia Areata
c. Pediculosis
d. Tinea Capitis (also called gray patch)

58. A young couple wants to adopt abstinence as contraception. Assuming the female is having a regular
monthly cycle of 28 days, you would advice them to avoid sex on AOF days except:
a. Day 4 - 17
b. Day 8 - 17
c. Day 10 - 19
d. Day 12 - 19
e. Day 17 - 21

59. Most common cause of cholecystitis in Australia


a. Gall stone at out flow
b. Gall stone in CBD
c. Pancreatitis
d. Tumour
e. Hepatitis

60.WOF is expected to happen after splenectomy for spherocytosis? (Splenectomy (surgical removal of the spleen)
cures the anemia of spherocytosis. Although the abnormal cell defect persists, the red cell life span returns to normal.)
a. RBC life span improves
b. Leukopenia
c. Reduction in anaemia (???? Dose it cure the amemia or improves its )
d. Reduction of spherocytes
e. Reduction in fragility of RBCs.

61.What is the cause of majority of HbsAg patients in Chinese population?


a. Blood product
b. IVDU
c. Child birth
d. Sexual spread (most common cause in westeners)
e. Haemophilia

62. A 60 years old female patient of RA takes 5mg of Prednisolone tabs BD for last 10 years. Now she
comes to you complaining sudden onset of pain and swelling of her right knee, which is warm and tender.
Your management would be:
a. Increase dose of NSAID
b. Decrease dose of NSAID
c. IV Antibiotics & hospitalisation
d. Knee X Ray for unrecognised trauma
e. Knee aspiration for cytology. (Septic Arthritis AMCQ ME Q29)

63. A 12 year old is having pain in his hip. His mother noticed a limp. He is afebrile, and the general
examination is normal. What is the most probable cause of his symptoms?
a. Slipped upper femoral epiphysis (is most common in growing children, especially between ages 11 and 15., more
common in boys, in children who are obese, and in children who are growing rapidly.)
b. Perthes disease (most commonly is seen in persons aged 3-12 years, with a median age of 7 years.)
c. Non specific synovitis
d. Osteomyelitis (usually occurs in long bones)
64. Photo of viral wart on the eyelid of a young female.
a. Wart
b. Molluscum contagiosum
c. Herpes simplex
d. Skin tag

65. AOF cause gynecomastia, except:


a. Spironolactone
b. Digoxin
c. Methyldopa
d. Labetolol

66. A 41 week primi comes to you complaining of no fetal movements for last 24 hrs. FHS is 140/mt, CTG
is normal, and rest of the examination is normal. You send her home. Next day she rings you stating that
there is again no fetal movement for the last 24 hrs. What will be you next step of management?
a. Tell her that all her examinations are fine, & she should not worry
b. Ask her to wait for another 24 hrs.
c. Admit labour ward and induce labour.
d. Immediate LSCS

67. Picture of supracondylar fracture (AMC Book): A young girl fall on her out stretched hand. What will
be your management?
a. Neurological study to check the Median Nerve involvement
b. Fasciotomy
c. # Reduction and assessment of circulation
d. Back slab and review in 24 hours

68. Many people attended a dinner party on a week end. 10% had diarrhoea. Most of them recovered
spontaneously, but few needed hospitalisation for severe dehydration. What is the most probable cause?

a. Giardia (It can be contracted by drinking water from lakes or streams)


b. Salmonella (Salmonella enterocolitis can range from mild to severe diarrheal illness. The infection is acquired through
ingestion of contaminated food or water.)( The incubation period is 8 to 48 hours after exposure,)
c. Clostridium (causes tetany)
d. Shigella (C/P and severity is very similar to salmonella but the incubition period is 1 to 7 days, with an average of 3 days.)

69.Complication of # of epiphyseal plate


(The most frequent complication of a growth plate fracture is premature arrest of bone growth.)
a. Retardation of longitudinal growth
b. Joint stiffness
c. Malunion
d. Non-union
e. Avascular necrosis

70.4 day old full term baby normal at birth. Suddenly collapse at cot. O/E baby is peripherally cyanosed ,
no pulse and respiratory distress. Diagnosis?
a. Pulmonary Hypertension
b. Fallot’s Tetralogy
c. PDA
d. CHD
e. Lt Heart hyperplasia.
f. Transposition of great vessels

---------------------------------------------------------------------------------------------------------------------------------
-------------------------------------------------------------------------------------------------------------------

MCQ 2001 May RECALL QUESTIONS

1. In which of the following diabetic ketoacidosis most commonly presents with (Diabetic ketoacidosis may lead
to the initial diagnosis of type 1 diabetes” IDDM”, as it is often the first symptom that causes the person to come to medical attention.
It can also be the result of increased insulin needs in someone already diagnosed with type 1 diabetes. Infection, trauma, heart attack,
or surgery can lead to diabetic ketoacidosis in such cases.)
1. Undiagnosed IDDM
2. Undiagnosed NIDDM
3. Known IDDM when stopped Insulin
4. Known IDDM with foot infection

2. Which is correct for NIDDM


1. Genetic factor is more important in IDDM then NIDDM (Family history and genetics play a large role in type 2
diabetes.)
2. They will never require insulin
3. 10-20% of IDDM need hypoglycaemic agents
4. Abdominal fat is the risk factor for NIDDM

3. Papillary necrosis Except (Renal papillary necrosis is most commonly associated with analgesic nephropathy, diabetic
nephropathy, renal transplant rejection, urinary tract obstruction, kidney infections , chronic alcoholism and sickle cell anemia. Sickle
cell anemia is a common cause of renal papillary necrosis in children.)
1. Analgesic nephropathy
2. Alcohol Nephropathy (bethhoven
3. Medullary sponge kidney (is a developmental abnormality occurring in the medullary pyramids of the kidney. MSK is
characterized by cystic dilatation of the collecting tubules)
4. Diabetis

4. Picture - girl Acne and hirsutism on the face. Presented for the first time (oral contraceptives, low-dose
glucocorticoids, and antiandrogens, should be chosen according to the patient's symptoms and source of androgens and should be
combined with traditional therapy for the specific dermatologic disorder.)
1. Antibiotics
2. Antibiotics and retinoids
3. steroids cream
4. antiantigen(Danazol) (androgen: Used in the treatment of endometreosis, menorrhagia, and cyclic breast pain)(OG-55)
5. cypropterone acetate (antiandrogen)

5a. Picture with 3 lesions- with raised border


1. Granuloma annulare
2. Erythema multiforme
3. picture -AMC book - leision on the face
4. Mycrosporum canis

5. What is not true about parkinsons disease (clinical oxford 383)


1. can stop while walking involuntary
2. tremor at sleeping
3. rest tremor
4. affect one side more than the other

6. Mycoplasma pneumonia not true


1. severe cough (Cough is usually dry , without phlegm or blood)
2. high fever
3. pleuratic pain
7. Which of the following drugs causes hypertension when stopped abruptly.
1. clonidine
2. metoprolol
3. methyldopa
4. ACE inhibitors

8. Regarding serum lipid profile and diet what is true


1. olive oil will increase LDL/HDL rato (increase HDL thus decrease the ratio)
2. positive energy bances increase LDH
3. Increse in dietry sturated fatty acids will increase VLDL
4. Fish 100 300gm per week will have no effect on plasma lipid(help reduce cholesterol)
5. Large alclhol intake can reduce LDL

9. Polycythemia rubra vera


1. clinical cyanosis may be difficult to detect
2. erythropoetin levels supressed (but is raised in secoundry polycythemia as in tumour secreting eryhtropoiten)
3. ESR
4. Sleap Apnoea may cause the problem

12. Which of the most common antibiotic- which cause pseudomembranous colitis fghgfghfg
1. Metronidazole
2. Ampicillin

13. Myasthenia gravis What is correct?


1. thymectomy is only indicated for thymoma (required if there is a thymoma because of the risk of local infiltration. Also
occasionally undertaken in other non-thymoma patients with myasthenia)
2. EMG can always positive (Standard EMG results are usually normal).
3. Some body does not have antibody (The antibody titre is raised in 90% of patients with generalised disease and in
approximately 60% of those with the ocular form)
4. Anticholinegic is the treatment (cholinesterase inhibitor is the treatment)

14. Hepatitis Cmost commonly - in Australia (Currently in Australia, the greatest risk for the transmission of hepatitis C is
through blood-to-blood contact involved in the sharing or re-using of injecting equipment such as needles and syringes)
1. Drug user
2. Homosexual
3. Heterosexual
4. Blood products

15. What are the side effects corticosteroids Except


1. Increase WBC
2. Neutropenia
3. Osteomalasia

16. Eradication of helicobacteria pylory(A short course of standard ulcer treatment usually heals ulceration but about 85% of
patients relapse within a year. In patients with duodenal ulcer, eradication of H. pylori causes the relapse rate to fall to 0 - 20%..)
1. incresed healing ulcer
2. decreased relapse rate

17. As a public health officer what is the most sensitive screening test -for lead poisoning (A guide to the
severity of the lead poisoning is given by blood lead levels. Lead particularly affects the enzymes involved in haem synthesis; thus a
screening test for early lead poisoning is the measurement of haem precursors, for example the free erythrocyte protoporphyrin.)
1. do an environmental histry study
2. blood lead level(A simple finger prick blood test is used to screen for lead poisoning.)
3. physical examination of every child

18. A pregnant women fixed and wide spliting second heart sound
1. ASD
2. VSD
3. Pulmonary stenosis
4. Coarctation of aorta

19. A 55 y.o. man presents with low motor neuron symptoms and depressed brachial rexlexes . Also he has
a sympromes of upper motor neuron at his legs.What is the most likely diagnosis? AMC book
1.motor neuron disease
2.cervical disc prolaps

20. 54 years old man with suddend on set left side weekness without disphasia. Possible diagnosis.
(Dysphasia occurs in cortical ishemia)
a. Right mid C artery
b. coratic arterry oclusioon
c. epilepsy
d. Internal capsule

21. 30 years old lady presenting with cheek pain and sensory loss, Dx?
a. trigerminal neuralgia (no sensory loss)
b. multiple sclerosis

22. G6PD deficiency, causes except


a. Moth ball
b. Ampicillin
c. Broad bean
d. Sulfernamide
e. Fioranturin

23. kerotoakathosis, correct


it usually growth quickly and spontaneus resolve

24. A hypertension patient with sudden onset pain behind left eye, with ptosis, palsy signs of VII, V, VI,
(paralysis of palate, eye movement paralyss), Dx?
a. aneurysm of posterior communication william ring (located in the midbrain and associated with sever headache)
b. midbrain infarct (has cranial nervees 3 and 4)
c. brain stem infarctment

25. Indication for thrombolytic therapy ?


a. new LBBB (Left bundle branch block (LBBB), unlike right bundle branch block, is always an indication of heart disease, most
commonly coronary artery disease)
b. new RBBB
c. Q waves (is an indicator of myocardial infarction)
d. T inversion (appears after infarction when the ST segment returns to normal but is also the result of other causes and could be
found in normal people)
e. ST depression (usually seen in ischemia rather than infarction)

26. a men presenting with chest pain mimic ischimic chest pain with prolonged period, which is correct?/
a. heparin and iv nitrate are not indicated
b. thrombolytic Tx can not given if ECG is normal(if chest pain is related to infarction)
27. A young man with cough, Mantoux +, CXR showed inacticve TB lesion, Mx?
a. triple treatment for 6 months and repeat CXR(culture of the sputum may take 4 to 7 weeks to provide a result; a
further 3 weeks is required to identify drug sensitivity)
b. isoniazid tx fro 3 mons and repeat CXR
c. sputum and await
d. repeat CXR in 3 months

28. a lady with diarrhoea, anaemia, fatigue, MCV 110 `


(Gastrointestinal causes of Macrocytosis include :stomach removal surgery, celiac disease [sprue], Crohn's disease)
a. pernicious anaemia **
b. gluten-sensitive enterotomy
c. crohn’s disease

29. Blood gas ph 7.45 (N 7.35-7.44), pco2 27, po2 65, hCO3 23,
a. Respiratory alkalosis with gas change impairment
b. Respiratory alkalosis without gas change impairment

30. COPD which is the risk of right heart failure


a. FEV <1 L
b. Diffusing Capacity <50%
c. Reduced Po2 (HypoxiaPolycythemia + Airway remodelling Pulmonary hypertension RHF)
d. Increased Pco2

31. COPD patient was brought in by ambulance. Was given 10 L O2 on the way. ABG Pco2 65 Po2 60 and
Hco3 23.
a. reduce O2 and redo the ABG 30 mins later qwqwqwq

32.Infective Endocarditis asssociated with


a. Vasculitis is the presentation of endocarditis (prolonged fever is the presentation)
b. staphal is the common cause (Streptococcus viridans is the most common)
c. it is the direct from rheumatic fever

33.The patient presents in the emergency department with BP-90/50


Pulse rate 98 min. CVP is 0.5 water. What is the possible cause? (Normal CVP is 2-6 mm Hg.)
1.Cardiac arrest
2.Hypovoluemic shock
3.Bowel obstruction
4.Pancreatitis

34.The patient presents with the severe pain which is radiating to the back, BP-180/95. What is the feature
you are going to find on the CXR:
1.Widened mediastinum ( A sign of aortic aneurysm)

35.The abattoir worker is presented with jaundice, fever, and malaise.


What is the possible DS: (Q fever could also occure in abattoir workers)
1.Leptosperosis (caused by exposure to the bacteria, which can be found in fresh water contaminated by animal urine, therefore it
occure in farmers, ranchers, abattoir workers or fresh water swimmers. But unlike brucellosis it is presented with Sever jaundice)
2.Brucellosis (Transmission of the disease to humans occurs by contact with infected meat,)
3.Malaria
4.Dengue fever (Dengue fever is a viral illness transmitted by mosquitoes.)

36.Chronic diarrhoea for 3 months could be caused by except:


1.Campylobacter Jejuni(There is an incubation period of 2-4 days before symptoms occur. Symptoms generally last one week)
2.Crohn’s disease
3.Ulcerative colitis
4.Giardiasis
5. Laxative abuse

37.Most common cause perfuse bleeding in elderly:


1.Diverticulosis (sudden and painless profuse rectal bleeding)
2.Cancer of the colon (Occult blood)
3.Crohn disease (bloody diarrhea is not common)
4.Ulcerative colitis (occult blood)
5. ischemic colitis (Bright redblood in the stool associated with sever abdominal pain)

38 The patients presents with oedema of the face .Mx: treatment for hypertansion,gaaut and alopurinol
,indometacin.The most possible cause of his oedema: (Indomethacin and other NSAIDs modulate IgE-dependent late
phase oedema in human skin)
1.Indometcin intoxication(stop the treatment) (angio-edema)
2.Renal failure (there are no urinary signs)

39.IgA nephropathy

40. a lady with oligio and creptitaion, poor prognosis is?


a. Haematuria is the poor prognosis

41. caute and chronic renal failure?


a. ultrasound

42. increase transferrin seen in:


a. haemochromatosis

43. A patient with COPD , given O2 10L/min, become unresponsive, which of F is possible blood gas
a. ph7.15, Po2 65, pco2 100 (Acidosis)

44. patient with heart failure and gout, treated with thiazide diuetics, ACE -or, alloperidol and
indomethocine. Test : K7, Na 1.25
a. Na low indicate Na depletion

45. Na of 122 indicates?


a. low plasma osmolority.

46. a long list of blood film


a. multiple myoloma

47. Indication of speletomy


a. chronic ITP

48. Asthma, all except


a. total lung capacity reduced.

49. indication for posterior column lesion?


a. Romberg’s test positive
50. Hypothyrodism, except?
a. small hand muscle atrophy

51. opening snap indicate?


a. mobility

52. What is not characteristic of Autosome recessive (25%)


a. 50/50 chance of inheriance (This is a sign of Autosomal dominant)

53. DM with autoneuropathy all except


a. urine incontinence (retention)
b. diarrhoea
c. bradycardia

54. Most reliable Dx for Giardia


a. stool exam of cyst (repeated stool examination maybe negative)
b. duodenal fluid

55. Coeleic Dis Dx to confirm?


a. Duodenal biopsy
b. Antibody-gliadin
c. Antibody-endomyial

56. infectious mononucleosis, all correct except,


a. is usually associated with hepatitis (Elderly patients with EBV present clinically as having anicteric viral hepatitis.)
b. Diagnosed by antigen detection (monospot test: detects the presence of heterophile antibodies)

57. angioplasty and stent comparing, which is correct?


(When angioplasty is performed alone, blockages can recur, although most of these arteries can be opened again successfully. This can
also occur when a stent is placed in the artery at the time of the angioplasty.)
a. reduce restenosis but same complications as angioplasty
b. more complications than angioplasty alone
c. no difference

---------------------------------------------------------------------------------------------------------------------------------
-----------------------------------

which of the following usually causes irreversible dilated cardiomyopathy?


(include nutritional deficiencies, valvular heart disease, anemia, stress, viral infections (rare), alcoholism (alcoholic cardiomyopathy),
and coronary artery disease. )
a) Endocrinopathies (e.g thyrotoxicosis)
b) metabolic disorders ( e.g hypophosphatemia)
c) prolonged ethanol abuse
d) high-dose doxorubicin therapy
e) aspirin overdose

2) The systolic ejection murmur in hypertrophic obstructive cardiomyopathy is diminished when a patient
does which of the following?( The murmur decreases with any increase in preload (eg, Müller maneuver, squatting) or increase
in afterload (eg, handgrip). The murmur increase with any decrease in preload (eg, Valsalva maneuver, that elicited by nitrate
medications, diuretics, standing) or with any decrease in afterload (eg, that elicited by vasodilators).
a) performs valsalva maneuver ( decrease preload)
b) lies down (increases the preload)
c) inhales amyl nitrate ( decrease preload)
d) stands up ( decrease preload)
e) is treated with digoxin (Digoxin should be avoided in HCM patients because of its positive inotropic effect.)(increases the
murmur)

ANS
Etiologies of DCM:
1. Idiopathic
2. Alcohol
3. Inflammatory (myocarditis)
4. Collagen Vascular Dse (SLE, PAN, etc)
5. Infectious (post viral, Chaga's, Lyme's, etc)
6. Neuromuscular (Duchenne's, Friedreich ataxia, etc)
7. Metabolic (uremia, nutritional, etc)
8. Endocrine (thyrotoxicosis, DM)
9. FAMILIAL
10. Peripartum
11. Toxic (cocaine, heroin, etc)
12. Radiation induced
13. Chemotherapeutic agents (doxorubicin or adriamycin)

Choices A-C are all reversible once the condition is treated and controlled or the intake of the offending
agent is stopped. So the best choice would be D.

The ff increases obstruction (increased murmur) of HOCM:


1. inotropes, vasodilators, diuretics
2. hypovolemia
3. tachycardia
4. squat to standing
5. valsalva
6. amyl nitrite inhalation

The ff decreases obstruction (decreased murmur) of HOCM:


1. negative inotropes
2. vasoconstrictors (increases the preload)
3. volume expansion (increases the preload)
4. bradycardia
5. stand to squat (increases the preload)
6. sustained handgrip(increases the afterload)

Looking at the choices given, B would be the nearest plausible choice

---------------------------------------------------------------------------------------------------------------------------------
-----------------------------------

1.Thyroid scan. Cold lesion. Which symptoms would you expect?


A. Symptoms of thyrotoxicosis
B. Single lump on the neck (thyroid cancer)
C. Myxedema

ANS:B
Cold nodule means non-functional node. Thyrotoxicosis may be a/with hot nodule. Myxedema is clinical
manifestation of hypothyroidism. Solitary node--> mostly benign, cancers in 20%--> both cold.
2. What is initial cardiac changes in the first hours of the newborn, except
A. Closure of the ductus arteriosus
B. Increase right ventricular pressure
C. Closure of the ductus venosus
D. Increased pulmonary pressure from right ventricular pressure

ANS:??A Ductus arteriosis definitely doesn’t close in few hrs.

Closure of the ductus arteriosus---> 10 to 21 days


RVP ---> decreases
Closure of the ductus venosus-> by 7thday flow disappears, doesn’t CLOSE immediately.
Pulmonary pressure falls, doesn’t increase. RVP decreases.

3. X-ray of the wrist. Scaphoid bone FRACTURE.


A. Plaster immobilisation including a thumb and a wrist
B. Plaster immobilisation including a thumb, a wrist and elbow
C. It will result in avascular necrosis of distal pole
D. Internal fixation is best treatment because of malunion
E. Crepe bandage from elbow to the wrist

ANS: A
A: correct ans
B: Incorrect- it is a below elbow cast.
C: Incorrect- proximal end gets avascular necrosis
D: Incorrect- Internal fixation indicated if- i) widely displaced fractures ii) a/with carple instability.
E: Incorrect.

4. Abdominal X-ray.
A. Early small bowel obstruction
B. Late small bowel obstruction
C. Early large bowel obstruction
D. Late large bowel obstruction

ANS- ?? Where is the X_ray?

5. ECG. An old lady complains of palpitations. BP=90/70. 150 pulse rate and
Atrial flutter?
A. Verapamil
B. Lignocaine (broad complex tachycardia)
C. Amiodarone

ANS: A
No hemodynamic compromise yet. So rate control with A is good. Lignocaine for VF. Amiodarone for drug
cardioversion.

6. ECG. 38 year-old male complains of severe central chest pain.


A. Acute inferior MI (Nausea and/or abdominal pain often are present in infarcts involving the inferior or posterior wall.)
B. Left vetricular ischemia

ANS: Can’t say! Both possible.


7. In which of the following conditions topical steroids are not useful
A. SLE (only systemic)
B. Atopic dermatits
C. PSORIASIS
D. Perioral dermatitis

ANS: A
Dermatitis--> used. Perioral--> lowest strength, shortest duration. Psoriasis--> used.

8. Eczema in infant. What is true?


A. Mostly presented as a nappy rash (they occur in the face and extensor aspect such as elbow and knees)
B. Ig E are not detected (mediated by IgE and type IV hypersensitivity reactions also known as Delayed hypersensitivity
reactions)

ANS: ??
Atopic eczema of Infancy: acute & involves face and trunk, napkin area usually spared, IgE present. Irritant
eczema--> nappy rash seen.

What do you guys say?

9. Hepatitis C is common, except


A. South-East Asian origin
B. Homosexuals (needs a gush of blood as in transfusion or organ transplantation to be transmited)
C. Renal dialysis

ANS: B
Mostly blood borne.

10. In epidemiological studies related to schizophrenia (SCH), the point-prevalence means: (An epidemiological
term that refers to the number of new cases of a disorder that appear in a population during a specific period of time.)
A. Current cases of SCH in a certain population group
B. Previous cases of SCH in a certain population group

ANS: I have no idea. Somebody please help.

11. Use of ACE inhibitors, except


A. Acute MI
B. One of the first choices in the treatment of essential hypertension
C. Hypertension in DM (drug of choice)
D. Aortic stenosis (as well as renovascular disease
E. Cardiac failure

ANS: D

12. Mycoplasma pneumonia


A. Caused by mycoplasma p…. (not pneumoniae)
B. Often associated with headache and tonsillitis
C. Usually diagnosed by culture (by serology and presence of cold agglutinin in serum)
ANS: B
Atypical orgn--> Routine Culture negative. Mild pneumonia with dry cough and constitutional symptoms.
---------------------------------------------------------------------------------------------------------------------------------
----------------------------------
AMC RECALL EXAM OCTOBER 2000

1. Hepatitis C in Australia mainly spread by

1. heterosexual (in China by Birth)


2. homosexual
3. intravenous drug users
4.blood products
5.

2. Anterior uveities most commonly seen in (The most common form of uveitis is anterior uveitis, which involves
inflammation in the front part of the eye, which is usually isolated to the iris. This condition is often called iritis. The inflammation
may be associated with autoimmune diseases such as rheumatoid arthritis or ankylosing spondylitis. Seronegative
spondyloarthropathies (SA), and mainly ankylosing spondylitis, is the most frequent cause of the disease.)
1. Rheumatoid Arthritis
2. SLE
3. Ankylosing spondilitis
4. .

3. Women presented with bleeding. She has a family history of bleeding (brother, father).
1. Haemophylia ((Hemophilia A and B are X-linked recessive disorders; therefore, they affect males almost exclusively))
2. Deficiency XII
3. Von Willebrand’s disease (A family history of a bleeding disorder is the primary risk factor for Von Willebrand disease. In
women with heavy or prolonged menstrual bleeding)
4. .
5. .Christmas disease (Is another name for haemophilia B)

4. Patient - pulmonary embolism (AMCQ ME Q42)


1. cyanosis
2. tachycardia
3. hypotension
4. haemoptysis
5.

5. Granuloma Anulare
1. premalignant (they are benign asymptomatic self limiting within 2 years)
2. itch
3. irregular raised edge (circular papule)
4. oral antifungal treatment (topical or intralisional corticosteroids)
5.

6. HLA typing most useful in


1. Renal Transplantations ( as well as bone marrow transplant.)
2. Cardiac transplantataions

6. For organ donation how to diagnose brain death


1. No EEG recording
7. Myelofibrosis
1. tear drop cells

8.Young man - Ferritin slightly elevated,


1. Acute blood loss
2. Heamoglobinopathy (example sidroblastic anemia)

9.Which does not transmit HIV


1. Cryopresipitate

10. Woman taking progesteron complaines of a headache in the afternoon. Paracetamol did not help.
1. stop progesterone (could be an early sign of stroke)
2. start ergotamine (Cluster headache begins during the early hours of sleep)
3. Relaxation method

11. Numbness of mouth . which nerve leision (Trigeminal Neuralgia.)


1. VII
2. V
3. II

12.
12. Woman (old) , history of tremor- increase when reach for an object. Treatment
1. propranolol (The beta-adrenergic receptor antagonist propranolol (Inderal) is used in the treatment of essential tremor.
Propranolol is most effective for upper limb tremor and less effective for head and voice tremors.)
2. levedopa and carbidopa (parkinsonism causes rest tremors)

13. Parkinsonism
1. loss of postural reflexes

14. women present with


1. cervical spondilosis

15. Which arthritis heals completely


1. Rheumatic fever
2. Rheumatiod arthritis

16. What type of renal calculi seen if urine is acidic


1. oxalate
2. uric acid
17. How to confirm gout
1. check synovial fluid for crystals

18. Asthmatic patient - which is not a proper management


1. taking prednisolone for every attack

19. Red painful eye dilated pupil


1. Glucoma

20. bulging eardrum -what is true


1. spontaneous rupture relives pain.

21. Spont. Pneumothorax


1. needle aspiration

22. Patient had a fall from a roof. BP -reduced, RR- inc


reased

23. In duodenal ulcer - not seen


1. Anorexia

24. cellulitis
1. group A streptococci

25. SLE
1. > 90% patients has DNA antibodies (ANA postive in 95%)(Anti-dsDNA postive in 70%)
2. > 50% has renal involvement (less than 50% have lupus nephritis)
3. .
4. .
5. .

26. Patient with Rheumatic fever developed a new murmur- lower end of the sternum
1. Aortic regurgitation (http://www.emedicine.com/EMERG/topic509.htm)
2. Pulmonary regurgitation
3. Rupture of

27. Patient with a carotid stenosis > 80%


1. with out surgery her chances of having a stroke in next 3 months is more

28. Phenytoin and warrfarin


1. if serum level -half of the needed therapeutic dose then double the dose
29. Hypercalcemia occurs ecxept
1. chronic pancreatitis

30. In Australia -why women live longer than men


1. genetics and biological reasons
2. use of more medicare
3. men are more violent

31. In Digoxin toxicity seen ecxept (Digoxin toxicity may cause any dysrhythmia)
1. nodular rytham (No P waves but normal QRS complex”Ventricular contraction” and normal rate)
2. ventricular trachacycardia
3. Atrial tachycardia

32. woman with R haemiparesis and loss of vision in the L eye


1.left anterior
2.left middle
4. middle cerebral

4.
5.

33. 70 years old woman -with 3 week history of confusion. Increased Phosphate, increased calcium.
1.paget disease (phosphate levels remain normal where as clacium increases du to immobilization)
2. renal failure (Salt, potassium, phosphorus, and other electrolytes may be restricted because the kidney cannot excret them)
3. .
4. .
5. .

34. Pituitary tumor


1. Bitemporal hemianopia
2. .
3. .
4. .
5. .

35. Emphysema - not true

1.
2.
3.
4.
5.

36. ITP

1. Extensor surfases
2. Splenomegaly (the splen takes up the platelests)
3. Rash (pinpoint red spots and petechial rash)
4. Haemorrage -fundoscopy
5. .

37. Mycoplasma Pneumonia


1. wrong antibiotic
2. .
3. .
4. .
5. .

38. Pressure gradient more than 50


1. you do surgery
2. .
3. .
4. .
5. .

39. Obstructive cardiomyopathy . RX


1.
2.
3.
4.
5.

40. All causing hypoglycemia ecxept


1. Prolactin inducing tumour
2. .
3. .
4. .
5. .

41. Meningitis complication except


1. Cephalohematoma
2. .
3. .
4. .
5. .

42. 34 year old women got 4-5 patches on the head


1. scraping under KOH
2. .
3. .
4. .
5. .

43. Pleural effusion in TB


1. lymphocytosis (Pleural fluid lymphocytosis, with lymphocytes greater than 85% of the total nucleated cells, suggests
tuberculosis (TB), lymphoma, sarcoidosis, chronic rheumatoid pleurisy, yellow nail syndrome, or chylothorax.)
2. redused sugar
3. transudate ( causes exudates not transudates)
4. .
5. .

44. man developed a rash on palms and the soles . Anal ------
1. second syphylis (The most common symptoms include skin rash, which can be varied in appearance, yet frequently involves
the palms and soles,)
2. yaws (Macular and hyperkeratotic lesions on the palms and the soles resembling syphilis may also be present.)
3. .
4. .
5. .

45. Atopic eczema except (bathe once-to-twice daily using mild soaps and dry quickly and immediately (within 3 min) lubricate
the skin. Creams are preferred over lotions, as they have lower or no water content. Acute attacks should be treated by mid-high
strength topical steroids for up to 2 weeks. reserve oral prednisone (at least 20 mg/d for 7 d) for the most severe cases)
1. use soap and water
2. .
3. .
4. .
5. .

46. Pulmonary Embolism

1. Tachypnoea
2. Cyanosis
3. Hypotension
4. .
5. .

47. In gross Acites


1. cannot palpate liver
2. .
3. .
4. .
5. .

48. In kernicteros
1. Increased in total bilirubin (total serum bilirubin levels ranged from 20.7-59.9 mg/dL)
2. Inreased in uncongugated bilirubin
3. Increased in congugated biluribin
4. .
5. ,

49. Obstructive cardiomyopathy RX

1.
2.
3.
4.
5.
50. All causing hypoglycemia ecxept

1. prolacting inducing tumour


2. .
3. .
4. .
5. .

51. Meningitis complication except

1. cephalohaematoma
2. .
3. .
4. .
5. .

52. L5 L4 affected
1.absent knee jerk
2. absent ankle jerk
3.
4.
5.

53. Acromegaly Ix
1. Oral and GH
2. .
3. .
4. .
5. .

54. All the following anomalies are corrected surgically except


1. fallots tetralogy (essentially surgical. Surgery is often in two stages with a palliative procedure to improve symptoms followed
later by a total repair)
2. patent ductus
3. VSD
4. Transposition of great vessels
5. .

55. Man present with a ureteric colic and brought a sample of urine which contained blood and asks for
pethidine. On examination NAD. Urine sample containes blood.
Rx.
1. take a another urine sample for blood
2. give him pethidine
3. .
4. .
5. .
56. patient presents with (night time) paresthesia and waisting of thenar muscles

1. carpal tunnel syndrome


2. ulnar nerve
3. .
4. .
5. .

57. to reduce staph. Infections


1. Rx all the carries
2. .
3. .
4. .
5. .
58. ACE inhibitors
1.increase afterload
2. increase preload
4. decrease afterload
5. decrease preload
5.

59. WOF cross the blood brain barrier


1. lipid soluble
2. low molecules
(The blood-brain barrier blocks all molecules except those that cross cell membranes by means of lipid solubility (such as oxygen,
carbon dioxide, ethanol, and steroid hormones) and those that are allowed in by specific transport systems (such as sugars and some
amino acids). Substances with a molecular weight higher than 500 daltons (AMUs) generally cannot cross the blood-brain barrier,
while smaller molecules often can.)

60. Patient Rx with warfarin now present with loss of consciousness.

1. extradural haematoma
2. subdural haematoma

61. Vascular dementia

62. Diabetic patient taking morning and evening doses. He wakes up at night with sweating
1. reduce evening dose

63. woman with ulcerative colitis


1.SCLEROSING CHOLANGITIS

6. Women presented with bleeding. She has a family history of bleeding (brother, father).
6. Haemophylia ((Hemophilia A and B are X-linked recessive disorders; therefore, they affect males almost exclusively))
7. Deficiency XII
8. Von Willbird (A family history of a bleeding disorder is the primary risk factor for Von Willebrand disease. In women with
heavy or prolonged menstrual bleeding)
9. .
10. .Christmas disease (is another name for haemophilia B)

64. same pt.going for surgery give


1. give factor VIII

65. “De ja vu” experience in

---------------------------------------------------------------------------------------------------------------------------------
----------------------------------

1. All of the following are true about duodenal ulcer, EXCEPT:


a) Hunger pain (food releive pain)
b) Loss of appetite
c) Weight gain (weight is normal)
d) Relapse and remissions
e) Reversed by antacids

2. A patient presents with a history of 2 episodes of Left eye blindness, Right hemiplegia, and transient
speech difficulty, which lasted 20 minutes each. Which is the most possible site of obstruction? (contralateral
hemiplegia, eye deviation toward the side of the MCA infarct, contralateral hemianopsia, and contralateral hemianesthesia. Eye and
head deviation toward the side of the lesion is probably due to damage of the lateral gaze center (Brodmann area 8)
a) Left middle cerebral artery
b) Right middle cerebral artery
c) Left anterior cerebral artery (causes numbness and weakness in the contralateral leg sometimes the arms)
d) Right anterior cerebral artery (causes numbness and weakness in the contralateral leg sometimes the arms)
e) Posterior cerebral artery (causes contralateral homonymous hemianopia only)

3. In which disease is anterior uveitis most commonly associated? (seronegative spondyloarthropathies (SA), and
mainly ankylosing spondylitis, the most frequent cause of the disease.)
a) Reiter’s disease
b) Ankylosing spondylitis
c) Syphilis
d) Gout
e) Rheumatoid arthritis

4. Which is the most common mode of transmission in Hepatitis C infection?


a) IV drug users
b) Homosexual active males
c) Heterosexual
d) Vertical Transmission
e) ?

5. Mycoplasma pneumoniae infection has which of the following characteristics, EXCEPT:


a) Cough is unusual (Worsening dry cough)
b) Pleural effusion is a characteristic feature (Pleural effusions can be seen rarely)
c) ESR is always high
d) Very severe headache
e) Responds to tetracycline treatment (and erythromycin)

6. A patient present with Haematuria. About 24 hours ago he had a sore throat. Which is the most likely
diagnosis?
a) IgA nephropathy (Recurrent Bloody urine may begin during or soon after a respiratory infection)
b) Post-streptococcal glomerulonephritis (may develop 1-2 weeks after an untreated throat infection, or 3-4 weeks after a
skin infection.)
c) Membranous glomerulonephritis
d) Nephrotic syndrome (Characterised by protinuria, albuminaemia and edema NO hematuria)
e) Acute pyelonephritis

7. Which of the following is true about Tuberculous Pleural Effusion ? (Pleural fluid lymphocytosis, with
lymphocytes greater than 85% of the total nucleated cells, suggests tuberculosis (TB), lymphoma, sarcoidosis, chronic rheumatoid
pleurisy, yellow nail syndrome, or chylothorax.)
a) Blood stained always
b) Acid-fast bacilli are always found
c) Most likely cells are lymphocytes
d) It is transudate (Exudate)
e) Glucose is normal or decreased

8. Acute attacks of asthma can best be treated with:


(The immediate treatment include: high flow oxygen (40-60%)-> nebulised ß2 agonist bronchodilators-> systemic
steroids 40-50 mg)
a) Inhaled Salbutamol
b) Oral steroids
c) Inhaled steroids
d) Oral theophylline
e) PEFR measured at home

9. A young man present with Pleuritic chest pain and cough. On percussion there is dullness over the right
base. There are no breath sounds in the right lower zone, but bronchial breathing is noted in the right
middle zone. Which is the most likely diagnosis?
a) Right lower lobe consolidation
b) Right lower lobe collapse
c) Pleural effusion
d) Right pneumothorax
e) Left tension pneumothorax

10. All of the following are true about Emphysema, EXCEPT:


a) Clubbing is present
b) Distant heart sounds
c) Diffusion capacity is decreased
d) Total lung capacity is increased
e) Hyperinflation of the lungs

11. Photosensitivity is present in:


a) Amiodarone treatment
b) SLE
c) Niacin deficiency (causes pellagra which is characterised four D's: photosensitive dermatitis, diarrhoea , dementia , death)
d) Thiamine deficiency (causes dry and wet beri beri. Its very common in alcoholics)
e) ?

12. In a patient with massive ascites, all are possible findings except:
a) Hepatomegaly( but difficult to palpate)
b) Shifting dullness
c) Eversion of umbilicus
d) Dullness to percussion
e) ?

13. Which of the following drugs can give Nephrotoxicity?( nephrotoxic effects of cyclosporine, aminoglycoside
antibiotics, cisplatin, amphotericin B, beta-lactam antibiotics and indomethacin)
a) Gentamycin (aminoglycoside antibiotics)
b) Gold
c) Digoxin
d) Sulphonamides
e) Mefanamic acid

14. A 65 year old woman became confused. Laboratory findings were; Calcium 4.8, increased Calciuria,
decreased Phosphate. Which is the most likely diagnosis?
a) Malignancy (osteolytic metastases) (it’s the most common and most serious cause of hypercalcemia)
b) Primary hyperthyroidism
c) Idiopathic hypercalcaemia/hypercalciuria (occur in a very young age)
d) Paget’s disease (increased plasma alkaline phosphatase, plasma calcium - usually normal)
e) Multiple myeloma(hypercalcaemia with low alkaline phosphatase levels (contrast this with the hypercalcaemia of bony
metastases)

15. Papillary necrosis is caused mostly by; (Renal papillary necrosis is most commonly associated with analgesic
nephropathy, diabetic nephropathy, renal transplant rejection, urinary tract obstruction, kidney infections , chronic alcoholism and
sickle cell anemia. Sickle cell anemia is a common cause of renal papillary necrosis in children.)
a) Diabetes
b) Alcohol
c) Analgesic nephropathy
d) Medullary sponge kidney
e) Sickle cell crisis ( common cause in children)

16. In a patient with acidic urine, which are the most commonly found stones?
a) Uric acid
b) Cystine
c) Calcium oxalates
d) Calcium carbonate
e) Xanthine

17. In patients with chronic renal failure:


a) Most patients develop osteoporosis
b) Bone dystrophy is present (characterised by: Osteomalacia, secondary hyperparathyroidism and vitamin D deficiency)
c

18. In rheumatic fever arthritis, which is the most typical finding? (migratory arthritis – Polyarticular involving
primarily large joints such as knees, elbows, ankles, and wrists, no residual damage and responds dramatically to salicylates)
a) Heals without deformities
b) Affects small joints equally as large ones
c) Is confined to one joint

19. Which of the following drugs most commonly cause prolonged QT interval?( drugs that were initially known
to prolong the QT interval were antiarrhythmics, and quinidine was the most commonly implicated agent.)
a) Digoxin
b) Procainamide (also causes QT interval but not as common as quinidin) http://www.ncbi.nlm.nih.gov/entrez/query.fcgi?
cmd=Retrieve&db=PubMed&list_uids=3751914&dopt=Abstract
c) Quinidine
d) Propranolol

20. A woman presents with dark colour urine, pruritus and jaundice. Blood examination shows a mild
increase in alkaline phosphatase, high AST and ALT. Which is the most likely diagnosis?
a) Viral hepatitis
b) Acute fatty liver
c) Cholangitis
d) Biliary obstruction with cholestasis
e) Cholecystitis

21. Which of the following is true regarding Phenytoin?


a) Should be given three times daily
b) It will decrease the effect of Warfarin (it increases the effect of warffarin)
c) It will increase the effect of OCP (it decreases the effect of OCP)
d) If given in combination with other drugs, the therapeutic dose should be halved
e) ?
22. Statistically women live longer than men. Which is the reason for this?
a) Females have a genetic advantage
b) Women use the medical system more often than men
c) Men have more stressful jobs than women
d) Men drive more aggressively
e) Women have less accidents

23. Treatment of hypertension is a proven factor in preventing:


a) TIA
b) Ischaemic stroke (Clinical Oxford page140)
c) Coronary artery disease
d) Congestive cardiac failure
e) Atherosclerosis

24. A patient with CRF and hypertension was dialysed. His BP improved after dialysis but he still had 2
episodes of high blood pressure. Which is now the initial step in your management?
a) Decrease protein in his diet
b) Ideal weight should be reached
c) Give Frusemide
d) Give anti-hypertensive therapy prior to dialysis
e) Give anxiolytics prior to dialysis

25. Hepatitis BV has never been found in which of the following blood products:
a) Fresh frozen plasma
b) Cryoprecipitate
c) Whole blood
d) Erythrocytes
e) White blood cells

26. In a woman with splenomegaly, WCC 33,000 and tear drop poikilocytes, which is the most likely
diagnosis?
a) CML
b) CLL
c) Hodgkin’s disease
d) Acute lymphoma
e) Myelofibrosis (progressive scarring of the bone marrow This causes blood formation to take place in other sites such as the
liver and spleen, causing enlargement. Charaterised by teardrop-shaped red blood cells.Its more common in females by a 2:1 ratio)

27. A woman, whose father and brother are haemophilic, has bleeding tendency. She has a history of
haemarthrosis and prolonged bleeding associated with dental procedures. Which is the most likely
diagnosis?
a) Haemophilia(Hemophilia A and B are X-linked recessive disorders; therefore, they affect males almost exclusively)
b) Christmas disease ( Is another name for haemophilia B)
c) Von Willebrandt’s disease (Von Willebrand's disease is a result of a variable deficiency of subunits of factor VIII which is
also deficient in haemophilia A. Bleeding may occur after surgery or when you have a tooth pulled. A family history of a bleeding
disorder is the primary risk factor for Von Willebrand disease. In women with heavy or prolonged menstrual bleeding)
d) Factor XII deficiency
e) Factor IX deficiency

28. Which of the following are true regarding Atopic Eczema?


a) Soap and water are helpful ( Are contraindicated they cause more irritation)
b) It is a contraindication for immunization
c) Flexure surfaces are commonly involved (The earliest lesions are often evident in the creases (ie, antecubital and
popliteal fossae), folowed by lesions in the cheeks and forehead and extensors of the lower legs,)
d) Positive family history of asthma and hay fever
e) Moisturizing agents are helpful (Treatment of weeping lesions may include soothing moisturizers, mild soaps, or wet
dressings.)

29. Which of the following is a feature of Parkinson’s disease?


a) Loss of postural reflexes
b) Hyperreflexia (superfical reflexes are normal , depp reflexes are interfered by rigidity)
c) Atactic gait (Parkinson’s is characterised by shuffling gait and festinant gait)
d) Resting tremor
e) Muscle weakness (normal muscle power with no weakness)

30. A diabetic patient is taking insulin twice a day. Before breakfast a combination of L/A and S/A and in
the evening one dose of L/A. He complains of sweating in the morning. What alteration in insulin doses
needs to be done? (Hypoglycemia )(somogyi phenomena)
a) Reduce dose of L/A in the morning
b) Reduce dose of S/A in the morning
c) Reduce dose of L/A in the evening
d) Increase dose of L/A in the evening
e) Increase dose of L/A in the morning

31. Which is the most specific antibody in SLE? (AMCQ ME-Q204)


a) dsDNA (Anti-DNA antibody sensitive but not specific; almost all patients with SLE have elevated serum ANA levels) .
b) ANA (positive in 95% of cases)
c) Anti Ro
d) Anti La
e) Anti LKM

32. A 24 year old student complains of 12 days of fever, malaise and sore throat. On examination he has
white tonsillar exudates, generalized lymphadenopathy, maculopapular rash on palms and soles and
excoriating lesions around the anus. Which is the most likely diagnosis?
a) Glandular fever (Infectious Mononucleosis:have a similar C/P exept for the maculopapular rash on palms and soles and
excoriating lesions around the anus which are found in yaws and secondery syphilis)
b) Syphilis (secondery syphilis)
c) Pemphigus (It is an autoimmune disorder characterised by blisters in the mouth, followed by skin blisters.)
d) Stevens Johnson syndrome (circular mucocutaneous lesions (target lesions)
e) ?

33. Which of the following is true regarding granuloma annulare?


a) Irregular raised margins (Annular margins)
b) Is pre-malignant (Bengin and self limiting within 2 years)
c) Caused by sun exposure (GA has traditionally been hypothesized to be associated with tuberculosis, insect bites, trauma,
sun exposure, thyroiditis, and viral infections, including HIV, Epstein-Barr virus, and herpes zoster virus. However, these suggested
etiologic factors remain unproven.)
d) Is autosomal recessive

34. A school teacher presents with patches of hair loss on scalp. Skin of the affected area is shiny. All of the
following are possible treatment except:
a) Nystatin ointment
b) Metronidazole
c) Ketoconazole
d) Griseofulvin
e) Nystatin cream

35. Which is the most important finding for the diagnosis of gout? (Synovial fluid analysis, Uric acid in blood and
Joint X-rays are the most important Inv)
a) Finding birefringent crystals (aspiration and examination of synovial fluid - shows negatively birefringent crystals)
b) Increased neutrophil count
c) Increased serum uric acid
d) Increased urinary acid
e) ?

36. In a patient with T8 pain and spastic paresis of lower limbs, which is the most likely diagnosis:
a) Herpes zoster
b) Spinal cord compression
c) Syringomyelia (It’s a very slow and progressive diasease that involves mainly the neck area)
d) Multiple sclerosis (Occure more commonly in women and are usually associated with cranial signs)
e) ?

37. Digoxin is indicated in all of the following except: (Digitalis glycosides are contraindicated in patients with
ventricular fibrillation or in patients with a known hypersensitivity to digoxin)
a) Atrial fibrillation
b) Atrial flutter
c) Atrial tachycardia
d) Nodal rhythm
e) SVT dye to AV nodal recovery

38. In a 30 year old patient with iron deficiency but no history f bleeding and whose ferritin level is
elevated, which is the next investigation?
a) Gastroscopy
b) Colonoscopy
c) Bone Marrow aspiration
d) Sigmoidoscopy (if cancer occurs in this area it will show gross blood not occult)
e) Haemoglobin electrophoresis

39. An overweight patient has had an episode of bulk diarrhoea with steatorrhoea. He has a history of
drinking alcohol for many years. Which of the following is the most useful investigation? (Steatorrhea occur in
cases such as coeliac disease, chronic pancreatitis and following a gastrectomy.)
a) IV pancreatography
b) Endoscopic pancreatography
c) Abdominal X-ray
d) Ultrasound (clinical oxford Page 252)(If normal consider CT/ERCP)
e) Faecal fat analysis

40. A young woman who takes oral contraceptive pills, complains of headaches over the last 3 weeks. The
frequency of headaches is five days per week. The headaches improve in the afternoon and during the
weekend. Which is the most appropriate management?
a) Stop OCP
b) Relaxation techniques
c) Ergotamine (Cluster headaches are sever and begin during the early hurs of sleep and last for only 2 hours)
d) Methysergide (Cluster headaches are sever and begin during the early hurs of sleep and last for only 2 hours)
e) Sumatriptan (Cluster headaches are sever and begin during the early hurs of sleep and last for only 2 hours)

41. Which of the following is the commonest organism that gives cellulitis?
a) Group A streptococcus
b) Group B streptococcus (common in infants)
c) Staphylococcus aureus
d) E Coli
e) ?

42. All of the following can cause glucose intolerance, except:


a) Pituitary tumour
b) Thyrotoxicosis
c) Acromegaly
d) Phaeochromocytoma
e) Pancreatitis

43. Which of the following is the most common complication of Infectious Endocarditis?
a) Mitral stenosis
b) Rupture of chordae tendineae
c) Aortic stenosis
d) Tricuspid stenosis
e) Aortic regurgitation

44. A woman present with tremor of the hands. Which of the following is correct? (essential tremor.)
(Propranolol is most effective for upper limb tremor and less effective for head and voice tremors.)
a) Disappears when she looks at her hands
b) Treatment of choice is Propranolol
c) Is present when she is on the phone
d) Disappears when she sleeps
e) Benztropine is helpful

45. A 35 year old policeman suffers from ulcerative colitis, and is treated with Sulfasalazine. He drinks 60g
of alcohol per day. Laboratory investigations show: serum bilirubin 18mmol/l; increased alkaline
phosphatase, GGT and SGOT. The most likely diagnosis is: (GGT may be high in liver disease. In particular it is a
feature of biliary outflow obstruction rather than hepatocellular damage.)
a) Fatty liver
b) Alcoholic cirrhosis
c) Biliary stones
d) Sclerosing cholangitis (PSC may precede the onset of ulcerative colitis (UC) or may develop following proctocolectomy.
raised serum alkaline phosphatase, raised serum aspartate aminotransferase in 90% of cases)
e) Cholestasis

46. Demential symptoms in an HIV positive patient can be due to all of the following EXCEPT: (Apart from
the ways in which HIV itself can directly affect the nervous system, infections and tumours can occur in the nervous system when
HIV damages the immune system. These include Cryptococcus, cytomegalovirus, Hodgkin's disease, neuropathy, non-Hodgkin
lymphoma (NHL), progressive multifocal leukoencephalopathy (PML) and toxoplasmosis.)
a) AZT side effects (HIV dementia became less common after the introduction of AZT)
b) Interferon side effects
c) Toxoplasmosis infection
d) AIDS dementia
e) Non-Hodgkin lymphoma

47. A patient with rheumatoid arthritis on long-term treatment with naproxen, came complaining of
fatigability over the past week. Lab analysis shows decreased Hb, Normal MCV, raised TIBC and ferritin.
Which is the most appropriate diagnosis?
a) Anaemic due to blood loss from GIT (presented as iron deficincy anemia)
b) Anaemia of the chronic disease (decrease erythropoietin)
c) Iron deficiency anaemia (Increase TIBC, Decrease ferritin, Hb and MCV)
d) B12 deficiency (Increase MCV)
e) ?

48. A lesion of spinal cord at L4/L5 level produces:


a) Loss of foot dorsiflexion (L5)
b) Absent ankle jerk (S1)
c) Loss of sensation of the medial aspect of calf and thigh (L3)
d) Absent knee jerk (L4)
e) Absence of sensation on the sole of foot (S1)

49. The most common cause of neck stiffness is: (Other causes: subarachnoid haemorrhage, tender posterior cervical
adenopathy, retropharyngeal abscess, rheumatoid arthritis, Phenothiazine toxicity, Tetanus, Upper lobe pneumonia)
a) Meningitis
b) Botulism
c) Phenothiazine toxicity
d) Tetanus
e) Pneumonia

50. A patient present with headache and neck stiffness over a 4 week period. All of the following are
possible except:
a) Extradural haematoma
b) Subdural haematoma
c) Lumbar puncture should be done ( Neck stiffness is a miningeal sign)
d) Urgent CT scan should be done
e) ?

51. Which of the following will decrease the gradient in hypertrophic cardiomyopathy?
a) Digoxin (+ve inotropic)
b) Verapamil (decreses the preload)
c) ACE inhibitors (decreses the preload thus increases the gradiant and murmur)
d) Prazosin (decreses the preload)
e) Propranolol (treatment with B-blockers is considered first-line therapy. By decreasing contractile force, B-blockers decrease
the outflow gradient and decrease oxygen demand. B-blockers also lengthen diastolic filling by slowing the heart rate)

52. Which of the following is true regarding aortic stenosis? (NADER CVS)
a) Sudden death is possible (Other causes of sudden death : Infarction, hypertrophy , cardomyopathy, myocarditis, WPW
syndrom, heart block)
b) Gradient of over 50% requires surgery (or valve size less than 0.5 cm2)
c) Angina suggest coexistent coronary artery disease
d) Doppler ultrasound is not accurate in diagnosis
e) Echocardiography may be used for diagnosis

53. A patient present with headache, prominent supra orbital ridges, prognathism, wide teeth spacing, thick
spade like hands and seborrhoeic, coarse, oily skin. Which of the following is the best investigation to
establish a diagnosis?
a) Insulin + glucose test
b) X-ray of the pituitary fossa
c) Cranial CT scan or MRI scan
d) Serum T4 + PRL + growth hormone test
e) Oral glucose tolerance test + GH ( Clinical Oxford Page 324)

54. A pregnant lady present with a widely split S2, cardiomegaly, systolic murmur best heard at pulmonary
area and a diastolic murmur in tricuspid area. Which is the most likely diagnosis?
a) Atrial septal defect
b) PDA
c) VSD
d) Pulmonary stenosis
e) Aortic stenosis

55. A 45 year old woman about to undergo cholecystectomy, has a history of bleeding tendency, increased
bleeding time, but a normal platelet count. Which of the following are indicated prior to surgery? (Von
Willebrand disease)
a) Vitamin K
b) Factor VIII concentrate
c) Fibrinogen
d) Fresh frozen plasma
e) ?

56. Which type of bilirubin can cross the blood brain barrier?
a) Unconjugated (Unconjugated bilirubin is lipid soluble and travels in the circulation largely bound to albumin. But the bilirubin
has to be free first in order to cross the blood brain barrier)
b) Conjugated (Conjugated bilirubin is water-soluble)
c) Biliary salts
d) ?
e) ?

57. A 28 year old female presented with a history of hypertension. Which of the following tests is best for
assessing the structure and function of her kidneys?
a) IV pyelogram
b) Renal ultrasound (Doppler)
c) Renal X-ray
d) CT scan
e) MRI

58. In a paraplegic patient suffering from overflow incontinence, which is the best management?
(neurogenic bladder)
a) Continuous catheterisation
b) Suprapubic catheterisation
c) Intermittent self-catheterisation (http://www.gpnotebook.co.uk/simplepage.cfm?ID=-1550188526)
d) Permanent cystostomy

59. A patient present with bilateral temporal hemianopia. Where is the lesion?
a) Pituitary fossa
b) Optic chiasm
c) Parietal lobe
d) Temporal lobe (superior quadrantanopia onj the opposite side)
e) Optic nerve
---------------------------------------------------------------------------------------------------------------------------------
----------------------------------

AMC RECALL PAPER: MCQ EXAM SERIES B, OCT 1999

Medicine
Type ‘A’ questions

CARDIOVASCULAR DISEASE

1. A patient with acute myocardial infarction used heparin; which of the following methods is used for
monitoring: (APTT is the most common test used to monitor heparin therapy.)
a. BT
b. PT
c. ARTT
d. INR
e. Fibrinogen

2. A patient has a mitral valve stenosis - all of the following signs are correct EXCEPT: (AMCQ ME Q37)
a. AF
b. S 1 increased
c. Palpitation increased S 2 in apex (diastolic shock)
d. S 3 ( s3 is caused by blood rushing into the ventricle before atrial contraction)
e. Presystolic murmur

3. In a patient with myocardial infarction was found a new systolic murmur on examination. Cardiac
ejection fraction was 55%. Which of the following is MOST probable cause:
a. Aortic regurgitation
b. Papillary muscle dysfunction
c. Mitral valve stenosis (earlysystolic murmur)
d. Papillary muscle rupture (causes mitral regurge which causes systolic murmur)
e. Tricuspid valve regurgitation

4. A young woman has hypertension with fibrosing stenosis of renal artery (60%) which of the following is
the MOST appropriate treatment: (Fibromuscular disease, a condition more common in young. Renal artery stenosis is among
the most common causes of secondary hypertension.)
a. Renal artery angioplasty
b. ACE Inhibitors (contraindicated in renal artery stenosis:decreaese Glomerular filtration rate)
c. Antihypertensives
d. Diuretics
e. Arteries dilation drugs

5. An obese patient with diabetes mellitus is under anti-hypertension treatment. His blood pressure is
160/100mmBg on examination. Which of the following is your INITIAL consideration for this patient:
(drug classes that may be used in combination with ACE inhibitors (first line in DM) or angiotensin II receptor antagonists include
beta blockers, long-acting calcium channel blockers or thiazide diuretics)
a. Decreased protein in his diet
b. Concurrent hypertensive therapy
c. Give diuretics (blood pressure needs to be controlled immediately)
d. Control sugar intake in the diet
e. Ideal weight

6. At which level of cholesteral you consider to give lipid-lowering statins (eg, simvastatin, pravastatin)
( Statins and dietary advice to lower serum total cholesterol by 20-25% or to reduce it to below 5.0 mmol/l, which ever would result in
the lower level. (Serum low-density lipoprotein cholesterol should be lowered by 30% or reduced to below 3.0 mmol/l, whichever
would result in the lower level.)
a. 6 mmol/l
b. 5.5 mmol/l
c. 5 mmol/l
d. 4.5 mmol/l
e. 4 mmol/l

7. Patient with coronary heart disease and xanthoma along the Achilles tendons. Which of the following is
THE MOST LIKELY diagnosis: (Xanthomas are deposits of fatty materials under the surface of the skin ranging in size from
very small to more than 3 inches in diameter. It occurs in conditions such as diabetes, primary biliary cirrhosis, some types of cancer,
and inherited metabolic disorders ‫ آ‬such as ‫ آ‬familial hypercholesterolemia. They most commonly apper on the elbows, joints,
tendons, knees, hands, feet, or buttocks.)
a. Familial hypercholesterolaemia
b. Familial combined hyperlipidaemia
c. Remnant removal disease
d. Hypolipoproteinaemia

8. Which of the following examination supports the diagnosis of pulmonary thromboembolism:


a. Chest PA X-rays (Usually apperas normal)
b. Pulmonary Doppler
c. Blood gas
d. Pulmonary ventilation perfusion mismatched on pulmonary scan
e. Lung function measurement

Contagious diseases

9. Which of the following is the MOST COMMON characteristic of pleura effusion of TB: (lymphocytosis is
the most common characteristic of pleural effusion)
a. Glucose decreased or absent (glucose - low in effusions due to rheumatoid arthritis, tuberculosis, SLE and malignancy)
b. Monocyte (lymphocytosis)
c. Blood stained (a characteristic sign of malignancy, pulmonary infarction or CVS diseases)
d. Protein <2g (Exudate: >3g/dl)
e. Find TB bacillus(acid-fast bacillus stains of pleural fluid are rarely diagnostic (fewer than 10% of cases)

10. Which following group is the MOST at RISK OF HIV infection:


a. Heterosexual
b. Homosexual
c. Intravenous drug user
d. Blood transfusion
e. Haemophilias

11. Which of the following group is LEAST LIKE of infection of HIV:


a. Heterosexual
b. Blood Transfusion (due to the screening methods avialable)
c. Homosexual
d. Haemodialysis
e. Haemophiliacs

13. A farmer has suddenly had undulant fever for 2-3 days with abruptly headache severe myalgia, jaundice
and petechial rash on the skin; liver and spleen enlargement. Which of the following is the diagnosis:
a. Brucellosis (Transmission occurs by contact with infected meat, the placenta of infected animals, or eating or drinking
unpasteurized milk or cheese. Symptoms begin with mild flu-like symptoms or with fever, chills, sweating, muscle aches (myalgia),
joint aches (arthralgia), and malaise. Classically, fever spikes occur every afternoon to levels around 104 degrees Farenheit.
"Undulant" fever derives its name from this undulating or up-and-down fever.)
b. Yellow fever (transmitted by mosquitoes, have the same C/P but no organomegaly or undulant fever)
c. Leptospirosis (similar clinical picture but fever is not undulant)
d. Malaria
e. Anthrax

14. Dengue fever, all followings are correct EXCEPT:


a. Arbovirus (its caused by four different arboviruses)
b. Mosquito transmission
c. Children get least severe illness
d. There is no specific treatment (It is generally self-limited and although uncomfortable is not fatal.)
e. Air droplet infection

15. A patient with mycobacteria infection which of the following is most appropriate treatment
a. cotrimoxazole
b. tetracycline
c. Amoxycilline
d. Metronidazole
e. Erithromycin

Emergencies
16. What is compatible with critical illness:
a. Increased cortisol , increased TSH
b. Both cortisol and TSH decreased
c. Increased cortisol, decreased TSH
d. Decreased cortisol, increased TSH
e. Normal cortisol, increased TSH

ENDOCRIN DISEASE

17. A 65 year old man has ‘bulk” diarrhoea with “oil”. He drinks alcohol for many years . Which of the
following is your investigation (Steatorrhea occur in cases such as coeliac disease, chronic pancreatitis and following a
gastrectomy.)
a IV pancreagraph
b Endoscopy pancreagraph
c. Abdominal X-ray
d Ultrasound (clinical oxford Page 252)(If normal consider CT/ERCP)
e Enema

18.For an elderly man, which above following blood sugar level need further investigation
a. 5 mmol/l
b. 5.5 mmol/l
c. 6 mmol/l
d. 6.5 mmol/l
e. 7 mmol/l

19.Side effective of corticosteroids including all the following EXCEPT (AMCQ ME Q 195, 196)
a Lymphocytosis
b. Lymphopenia
c. Hirsutism (Caused by inhaled steroids(beclomethasone)
d. Osteoporosis
e. Weight gain (Fluid retention)

20. A patient has headache, prominent supraorbital ridge prognathism teeth spacing increased,thick spade-
like hands and seborrhoea and coarse oily skin. Which of the following is BEST investigation to establish
diagnosis:
a. Insulin-glucose
b. X-ray of pituitary test
c. Cranial CT scan or MRI scan
d. SERUM T4+PRL+growth hormone level
e. Oral glucose tolerance test (OGTT) (Clinical Oxford) Page 324)

21.Which following patient is LEAST LIKELY to suffer primary hypothyroidism: ( Charaterised by menorrhagia
as well as Menstrual irregularity Generalized weakness Brittle or thinning hair Deep, husky voice Weight gain)
a. 65 year old female with goitre
b. 35 year old female with depression
c. 28 year old female with 3 years menorrhagia (too young)
d. 18 year old boy with relative less bone age
e. 32 year female with anaemia unresponsive to iron, B 12and folate (Anemia is due to decreased oxygen carrying
requirement.)

22 Patient has a single lump on one side of the thyroid, all following situation s suggest malignant
EXCEPT
a. Single nodule
b. US showed a solid nodule
c. Thyroid scan show “HOT’ lump
e. Associated with increased serum thyroglobulin (Thyroglobulin is a useful marker for tumor recurrence although they
are not helpful diagnostically because they are elevated in most benign thyroid conditions.)
f. Associated with hoarseness

DESEASE INVOLVING EYE

23. Uveitis is MOST COMMONLY found in which of th e following diseases: (The most common form of uveitis
is anterior uveitis, which involves inflammation in the front part of the eye, which is usually isolated to the iris. This condition is often
called iritis. The inflammation may be associated with autoimmune diseases such as rheumatoid arthritis or ankylosing spondylitis.
Seronegative spondyloarthropathies (SA), and mainly ankylosing spondylitis, is the most frequent cause of the disease.)
a. Reiter’s disease
b. Rheumatoid arthritis
c. Ankylosing spondylitis
d. Sjogren’s syndrome
e. Psorisis

24._____________________
a. An infection of frontal sinuses
b. Thyrotoxicosis
c. Neoplastic lesion in lamina cribrosa
d. Nasopharyngeal tumour invading the orbit
e. Glaucoma

GASTROINTESTINAL DISEASE

25. Eradication of HELICOBACTER PYLORI for duodenal ulcer: (Once the H. pylori bacteria are eliminated,
recurrence rates are very low in the United States.)
a. Increase ulcer healing rate
b. Influence relapse rate
c. Decrease rate of gastric lymphoma
d. Decrease local gastritis
e. Decrease cimetidine dosage

26 A young patient comes from overseas with diarrhoea, no blood. Temperature 37.9C, stool examination
showed few Salmonellas. What is your management: (The objective of treatment is to replace fluids and electrolytes (salt
and minerals) lost by diarrhea. )
a. Observation and repeat stool examination 3 days later
b. Broad spectrum antibiotic like amoxicillin
c. Trimethoprine plus sulphasalazine
d. Cotrimoxazole plus trimethoprim
e. Reassure

27. Which of the following is MOST RELATED to adenoma/carcinoma of the colon: (Dietary factors that have
been associated with colon cancer are a high-meat, high-fat, low-fiber diet.)
a. Aspirin can caused
b. Low fibre diet
c. Saturated fat more than the unsaturated fat in the diet (A large intake of polyunsaturated fat may increase the risk for
some types of cancer. On the other hand saturated fat is one of the major risk factors for heart disease.)
d. Alcohol
f. Smoking

28. A 28-year-old policeman on sulphasalazin therapy for ulcerative colitis, Right hypochondrial pain.
SGPT and alk, phosphatase increased, bilirubin mild increased, SGOT normal and liver aminotransferase
enzymes normal (?) which of the following is THE MOST LIKELY diagnosis: (The diagnosis of primary
sclerosing cholangitis should be suspected in a patient with ulcerative colitis who has abnormal LFTs, especially a raised alkaline
phosphatase)
a. Primary biliary cirrhosis
b. Side effect of sulphasalazin
c. Sclerosing cholangitis (PSC may precede the onset of ulcerative colitis (UC) or may develop following proctocolectomy.)
d. Cholangitis
e. Acute viral hepatitis

29. Patient with supposed hepatoma. Which of the following questions is MOST helpful for diagnosis:
a. Present liver cirrhosis
b. Alcohol liver disease
c. Acute hepatitis
d. Cholangitis (usually associated with cholangiocarcinoma)
e. Family history of liver hepatoma

30. Ascitis
a. _______
b. Bilateral abdominal varicosis
c. Peri-oral teleangiectasia
d. Jaundice and palmar erythema
e. Dupytren’s contracture

HAEMATOLOGICAL DISEASE

31. In anaemia patent with increased transferrin. All of the following is correct EXCEPT: (Saturation not TIBC)
a. Increased serum ferritin
b. Decreased serum ferritin
c. Increased total iron binding capacity
d. Increased transferrin
e. Decreased serum iron

32. An anaemic patient with increased transferrin. All following are correct EXCEPT:
a. Thalassemia major
b. Chronic disease (due to decrease protien synthesis)
c. Iron deficiency
d. Sideroblastic anaemia
e. Haemolysis (iron overload)

33. Which of the following is typically occurring in heparin - induced thrombocytopenia:


a. Joint bleeding
b. Petechial purpura on the skin
c. Thrombosis
d. Haematuria
e. Ecchymosis

34. In haemolytic anaemia, all is correct EXCEPT:


a. Increased urobilinogen
b. Serum urobilin
c. Erythrocyte count
d. Decreased MCV (normochromic normocytic)
e. Increased serum iron

35. A 67 year-old alcoholic man has backache, ESR 120mm/h. Which of the following is MOST LIKELY
diagnosis:
a. Multiple sclerosis
b. Acute lymphatic leukaemia
c. Hodgkin disease (Alcohol-induced pain at sites of nodal disease is specific for HD)
d. Non-Hodgkin disease
e. Multiple myeloma (rouleaux formation and a high ESR occur)

36. A patient with splenomegaly 12cm below ribs. There was no lymph node enlargement. Leucocytes
33000, decrease in erythrocytes, leukocytosis. What is the diagnosis:
a. Chronic granulitic (?) leukaemia (pancytopenia)
b. Myelofibrosis
c. Hodgkin disease (has lymphocytosis and splenomegaly but associated with Palpable painless lymphadenopathy)
d. Non-Hodgkin disease (has lymphocytosis and splenomegaly but associated with Palpable painless lymphadenopathy)
e. Acute lymphoma

37. A 67 year-old patient with long standing alcoholism. Lab: HB 9.8; ESR 120: leukocytes 10000 and left
shift. Which of the following is diagnosis:
a. Viral hepatitis
b. Multiple myeloma
c. Iron deficiency anaemia
d. Fatty liver

38. A 28 year-old female with long standing alcoholism, history of bowel irritation. Lab: HB 9.8; MCV
110. Which is the MOST LIKELY diagnosis:
(Gastrointestinal causes include:stomach removal surgery, celiac disease [sprue], Crohn's disease)
a. Whipple’s disease (affects middle-aged white men and is extremely rare, and risk factors are unknown.)
b. Colorectal carcinoma (patient is too young)
c. Coeliac disease(common in young alcoholic females)
d. Irritative bowel syndrome
e. Ulcerative colitis

IMMUNOLOGICAL DISEASE

39. A 74 year-old women have polymyalgia rheumatica. What is CHARACTERISTIC: (AMCQ ME Q34)
a. Distal limb muscle aching (proximal not distal)
b. Increased muscle CPK (normal)
c. There is fatigue, fever and depression
d. ESR normal (Increased)
e. Giant cell arthritis

NEUROLOGICAL DISEASES

40. Left homonymous hemianopia. Where is the lesion:


a. Right temporal lobe
b. Left optic nerve
c. Optic chiasm
d. Right optic radiation
e. Right occipital cortex
f.
41. A 64 year-old patient suddenly had face, arm and leg weakness on the same side of the body. No
sensory loss or loss of consciousness, where is lesion:
a. Middle cerebral artery (loss of cortical sensation)
b. Carotid artery occlusion (there is loss of concious and contralateral hemianasthesia)
c. Hypertensive disease local in the internal capsule (the sensory fibers realy the the thalamus to radiate to the cortex.
Thus sensation is usually spared but not always)
d. Infarction in the putamen
e. Aneurism of the sinus cavernosum (involves cranial nerve palsy particularly ocular manifestations)

42. All following signs are present in Parkinson disease EXCEPT:


a. Loss of postural reflexes
b. Myoclonus
c. Decreased reflexes (superfical reflexes are normal , depp reflexes are interfered by rigidity)
d. Muscle weakness (normal muscle power with no weakness)
e. Broad wide gait (Parkinson’s is characterised by shuffling gait and festinant gait)

43. Upper motor neuron damage - all can occur EXCEPT:


a. Hyperreflexia in arm
b. Spasticity of the extensor muscles of the leg (spasticity affects the extensors of the LL and the flexors of the UL while
weakness affects the flexors of the LL and extensors of the UL)
c. Proximal muscle weakness (weakness below the level of the lesion)
d. Absent abdominal reflexes (superfical reflex are absent while deep reflxes are exaggerated)
e. Ankle clonus (it signifies the hypertonia of an upper motor neurone lesion)

44. A 35 year-old female with facial pain, associated sensory delicate, no trigger zone. What is the
diagnosis:
a. Trigeminal neuralgia (characterised by trigger zones)
b. Ramsay - Hunt syndrome (is a neurologic disorder caused by varicella zoster, produce facial paralysis and rash either in the
ear, tongue, or palate.)
c. Bell’s palsy (no facial pain. Pain may only apear behind the ear)
d. Multiple sclerosis
e. Tolosa - Hunt syndrome (is a painful unilateral ophthalmoplegia)

45. These are all phenytoin side effects EXCEPT:


a. Nystagmus
b. Osteomalacia
c. Stimulate osteoporosis
d. Lymphoma
e. Deafness

46. Patient 64 year-old, suddenly occurred hemiparesis and paralysis of soft palate and Horner’s syndrome
on the same side of the body. Where is the lesion: (Horner’s syndrome can result from a stroke in the brainstem, injury to
the carotid artery, a tumor in the upper lobe of the lung, and cluster headaches.)
a. Middle cerebral artery (causes cortical strokes not brainstem strokes)
b. Carotid artery occlusion (horner syndrom occur in carotid arteriy dissection or injury but not insufficency as it is in close
relation with the third order neurons of the sympathetic nerve)
c. Vertebro-basilar artery (Lateral medullary infarct)
d. Giant cell arteritis
e. Subclavian steal syndrome

RESPIRATORY DISEASES

47. A 55 year-old patient has blood pressure 200/130mmHg. His left kidney is small and urine tests and
kidney functions are normal. What is your NEXT investigation: (computed tomographic angiography and MRA are
not reproducible or sensitive enough to rule out renal artery stenosis in hypertensive patients)
a. Intravenous pyelography
b. Retrograde
c. Renal CT scan
d. Renal scan
48. _________ and O2 _______was given___O2 therapy 4 hour later_______ showed PO2 68 mmHg, and
PCO 2 40 mmHg. What is your next therapy:
a. Reduce O2 concentration and do blood gas analysis after 30min
b. Give bronchodilators
c. Hydrocortisone
d. Intubation and ventilation
e. Antibiotics injection

49. Patient has sudden dyspnea. Physical examination found left upper chest respiratory movements
increased. In left lower part percussion dullness and decreased respiratory sounds. What is the diagnosis:
a. Atelectasis
b. Pneumonia (consolidation is asocaiated with increase respiratory sounds)
c. Pleural effusion
d. Pulmonary thrombosis(associated with acute chest pain and only deveolps consilidation later in the disease)
e. Pneumothorax (resonant allover)

50. Which of the following is the MOST consistent with the diagnosis:
a. Oat cell carcinoma-clubbing
b. Pneumonia-hyperresonance + decreased chest wall movements (consolidation=dullness)
c. Emphysema-bilateral alveolitis + clubbing (NO clubbing)
d. Fibrosing alveolitis-widespread respiratory??
e. Asthma-prolonged expiration with wheeze

RENAL DISEASES

51. A young patient has haematuria and proteinuria with respiratory infection. There is no hypertension. He
has similar situation before. The diagnosis is:
a. Nephrotic syndrome (Characterised by protinuria, albuminaemia and edema NO hematuria)
b. Streptococcal glomerulonephritis (may develop 1-2 weeks after an untreated throat infection, or 3-4 weeks after a skin
infection.)
c. IgA nephropathy (Recurrent Bloody urine may begin during or soon after a respiratory infection)
d. Acute pyelonephritis
e. Membranous nephropathy

52. Post-operative third day serum Na 165mmol/l and urine Osmo is 250 (normal 500 - 1000). The MOST
LIKELY cause is: (AMCQ SU-Q90)
a. Thiazide diuretic dose is too high
b. Diabetes mellitus
c. Diabetes insipidus
d. SIADH
e. Acute renal failure

53. Staghorn calculi can be found in the entire following situation EXCEPT: (Most common in women because of
increased incidence of urinary tract infection and 1° hyperparathyroidism. It is also known as triple-phosphate, infection, phosphatic,
and urease stones.)
a. Recurrent pyelonephritis (Struvite stones are invariably associated with urinary infections. Specifically, the presence of
urease-producing bacteria, including Ureaplasma urealyticum and Proteus (most common, )
b. Immobilisation
c. Idiopathic hypercalciuria
d. Primary hyperparathyroidism

AMC 23RD OCTOBER 1999


MCQ PAPER I
PAEDIATRICS
p.s : the following is a reconstruction of the essence only of the question and answer ( stem and answer
options) . The great majority of the actual exam questions were very much longer in the form detailed
clinical case histories.
R.?/? = repeated questions and when
R.T = repeated topic only
R.AMCQ = repeated question from “Annotated MCQ’ book

35Type ‘A”question:

Behavioural problem
1 A 7 yr with enuresis . What is the most likely cause? (In 99% of all bedwetting cases, the root cause is an inherited
deep-sleep disorder.)
a. UTI
b. Family disorder
c. O.M
d. Vesico -ureteric reflux
e. Ectopic ureters
CVS:

2 A 6 yr boy found to have a systolic murmur on the left sternal border. His weight is on the 3rd centile.
What is your advice to h is parents? (Spontaneous closure is common in children under a year old and rare over 2 years old.)
a. That he has Atrial Septal Defect
b. That he has Ventricular Septal Defect
c. That he should be referred to a cardiologist
d. That the murmur will remit spontaneously
e. That surgery is essential to correct his condition

Emergencies:
3 WOF is the most likely cause of meningitis in the 4 y.o child? (bacterial meningitis more often affects children while
viral meningitis more commonly affects young adults. It's mainly caused by the Haemophilus influenzae bacterium, and the
meningococcus type B and C bacteria.)
a. E.coli
b. Haemophilus influence
c. Adeno-virus
d. N. Meningitidis
e. Strep. Pneumoniae

4 A 3 y.o child ingested washing detergent . What is the most appropriated initial action
a. Give the child milk to drink
b. Urgent endoscopy
c. Induce vomiting by ipecac
d. Gastric lavage
e. Induce osmotic diarrhoea by mannitol

GIT

5 Jaundice 5 days after birth is most likely due to (physiological jaundice. This condition appears at two or three days old
and begins to disappear towards the end of the first week.)
a. Physiological deficiency of glucoronyl transferase
b. Physiological excessive destruction of the RBCs
c. Iso-immunization
d. Biliary duct atresia
e. Breast milk jaundice
6 A 2 y.o presented with chronic constipation. AXR shower heavy faecal loading of the colon . P/R was
uneventful. What is the most appropriated next step?
a. Double contract barium enema
b. Rectal biopsy (Hirschsprung's disease is a condition that usually manifests itself in the immediate neonatal period by the failure
of passage of meconium followed by obstructive constipation.)
c. Anal dilation
d. AXR
e. Laxatives

7A child brought in by his mother b/o vomiting. His weight is within normal centiles for his age. WOF is
the most PPROPRIATE?
a. Urine microscopy and culture
b. Advise the mother to make the milk formula thicker
c. Dilute his milk with water
d. Give him less milk
e. Check the strength of the milk formula

8 WOF is most likely cause of mouth ulcers in children?


a. Aphthous ulcer
b. Behcet’s syndrome
c. Herpes simplex
d. Herpes Zoster
e. Self inflicted

Haemo
9 A young boy presents with widespread petechial rash . Otherwise well Blood film shows the following :
Hb 10w , WCC 10w, Platelets 50x109 (150-400x 109)
WOF is the most likely cause?
a. TIP
b. Salicylate overdose
c. Anaphylactoid purpura
d. Heamophilia
e. Von willebrand’s Disease

Malignancies:
10 5 yo presented with a mass in the left lower abdominal quarant. Otherwise , he is well. What is the most
likely cause?
a. Neuroblastoma
b. Acute leukaemia
c. Nephroblastoma (Wilm’s tumour)
d. Hodgkin’s disease
e. Non-hodgkin lymphoma

11A child discovered a lump in the lower posterior triangle of the neck . O/E the lump fluctuated and
transilluminated. Its consistency was soft. What is the most likely cause?
a. Branchial cyst (Anterior triangle of the neck under cover of the anterior boarder of the sternomastoid between the upper third
and middle third)
b. Carotid body tumour(Anterior triangle of the neck, solid swelling, mobile horizontaly but not verticaly, transmited pulsation)
c. Enlarge lymphnode (most common cause of swelling in the neck but is firm and non fluctuating)
d. Thyroglossal cyst (Anterior triangle of the neck)
e. Cystic hygroma (Occur at the root of the neck in the posterior triangle at birth or in the early months, cystic transluscent
swelling that increases in size with coughing or cring)
Meonatology:
12 A full term neonate was born at 30/4, 1500 g was noticed to be cyanosed on the hands and feet . O/E the
infant was responsive and jittery on handing. The respiratory rate is 40/ min and lung and heart are
clinically normal . WOF is correct?
a. The infant most probably has a convulsion
b. O-2-- should be given
c. Blood sugar level should be checked
d. Blood calcium level should be checked
e. The infant’s rectal temperature should be checked

Neurology:
13All the following are true about cerebral palsy except: (AMC PA-Q7)
a. Epilepsy occurs in less than 50% of the patients
b. Feeding difficulties are common (AMCQ PA-Q4)
c. Majority develop mental handicap (IQ less than 70%)
d. Spastic diplegia is the most common type
e. Is associated with intellectual disability (dosnot cause it)
Resp:

14. A 1 year-old boy was brought to you c/o fever. O/E. you find his temp. to be 37.5oCwith widespread
bilateral wheezing on chest auscultation. Two family members just recovered from upper respiratory tract
infection. The MOST LIKELY cause is:
a. Acute bronchiolitis
b. Croup.
c. Asthma
d. Pneumonia
e. Inhaled foreign body

---------------------------------------------------------------------------------------------------------------------------------
---------------------------------
MCQ 2001 May RECALL QUESTIONS

MEDICINE

1. In which of the following diabetic ketoacidosis most commonly presents with (Diabetic ketoacidosis may lead
to the initial diagnosis of type 1 diabetes” IDDM”, as it is often the first symptom that causes the person to come to medical attention.
It can also be the result of increased insulin needs in someone already diagnosed with type 1 diabetes. Infection, trauma, heart attack,
or surgery can lead to diabetic ketoacidosis in such cases.)
1. Undiagnosed IDDM
2. Undiagnosed NIDDM
3. Known IDDM when stopped Insulin
4. Known IDDM with foot infection

2. Which is correct for NIDDM


1. Genetic factor is more important in IDDM then NIDDM (Family history and genetics play a large role in type 2
diabetes.)
2. They will never require insulin
3. 10-20% of IDDM need hypoglycaemic agents
4. Abdominal fat is the risk factor for NIDDM

3. Papillary necrosis Except (Renal papillary necrosis is most commonly associated with analgesic nephropathy, diabetic
nephropathy, renal transplant rejection, urinary tract obstruction, kidney infections , chronic alcoholism and sickle cell anemia. Sickle
cell anemia is a common cause of renal papillary necrosis in children.)
1. Analgesic nephropathy
2. Alcohol Nephropathy (bethhoven(
3. Medullary sponge kidney (is a developmental abnormality occurring in the medullary pyramids of the kidney. MSK is
characterized by cystic dilatation of the collecting tubules)
4. Diabetis

4. Picture - girl Acne and hirsutism on the face. Presented for the first time (oral contraceptives, low-dose
glucocorticoids, and antiandrogens, should be chosen according to the patient's symptoms and source of androgens and should be
combined with traditional therapy for the specific dermatologic disorder.)
1. Antibiotics
2. Antibiotics and retinoids
3. steroids cream
4. antiantigen(Danazol)
5. cypropterone acetate (antiandrogen)

5a. Picture with 3 lesions- with raised border


1. Granuloma annulare
2. Erythema multiforme
3. picture -AMC book - leision on the face
4. Mycrosporum canis

5. What is not true about parkinsons disease


1. can stop while walking involuntary
2. tremor at sleeping
3. rest tremor
4. affect one side more than the other

6. Mycoplasma pneumonia not true


1. severe cough (Cough is usually dry , without phlegm or blood)
2. high fever
3. pleuratic pain

7. Which of the following drugs causes hypertension when stopped abruptly.


1. clonidine
2. metoprolol
3. methyldopa
4. ACE inhibitors

8. Regarding serum lipid profile and diet what is true


1. olive oil will increase LDL/HDL rato (increase HDL thus decrease the ratio)
2. positive energy bances increase LDH
3. Increse in dietry sturated fatty acids will increase VLDL
4. Fish 100 300gm per week will have no effect on plasma lipid(help reduce cholesterol)
5. Large alclhol intake can reduce LDL

9. Polycythemia rubra vera


1. clinical cyanosis may be difficult to detect
2. erythropoetin levels supressed (but is raised in secoundry polycythemia as in tumour secreting eryhtropoiten)
3. ESR
4. Sleap Apnoea may cause the problem

12. Which of the most common antibiotic- which cause pseudomembranous colitis
1. Metronidazole
2. Ampicillin
13. Myasthenia gravis What is correct?
1. thymectomy is only indicated for thymoma
2. EMG can always positive (Standard EMG results are usually norma).
3. Some body does not have antibody (The antibody titre is raised in 90% of patients with generalised disease and in
approximately 60% of those with the ocular form)
4. Anticholinegic is the treatment (cholinesterase inhibitor is the treatment)

14. Hepatitis Cmost commonly - in Australia


1. Drug user
2. Homosexual
3. Heterosexual
4. Blood products

15. What are the side effects corticosteroids Except


1. Increase WBC
2. Neutropenia
3. Osteomalasia

16. Eradication of helicobacteria pylory(Once the H. pylori bacteria are eliminated, recurrence rates are very low in the
United States.)
1. incresed healing ulcer
2. decreased relapse rate

17. As a public health officer what is the most sensitive screening test -for lead poisoning (A guide to the
severity of the lead poisoning is given by blood lead levels. Lead particularly affects the enzymes involved in haem synthesis; thus a
screening test for early lead poisoning is the measurement of haem precursors, for example the free erythrocyte protoporphyrin.)
1. do an environmental histry study
2. blood lead level(A simple finger prick blood test is used to screen for lead poisoning.)
3. physical examination of every child

18. A pregnant women fixed and wide spliting second heart sound
1. ASD
2. VSD
3. Pulmonary stenosis
4. Coarctation of aorta

19. A 55 y.o. man presents with low motor neuron symptoms and depressed brachial rexlexes . Also he has
a sympromes of upper motor neuron at his legs.What is the most likely diagnosis? AMC book
1.motor neuron disease
2.cervical disc prolaps

20. 54 years old man with suddend on set left side weekness without disphasia. Possible diagnosis.
(Dysphasia occurs in cortical ishemia)
a. Right mid C artery
b. coratic arterry oclusioon
c. epilepsy
d. Internal capsule

21. 30 years old lady presenting with cheek pain and sensory loss, Dx?
a. trigerminal neuralgia
b. multiple sclerosis
22. G6PD deficiency, causes except
a. Moth ball
b. Ampicillin
c. Broad bean
d. Sulfernamide
e. Fioranturin

23. kerotoakathosis, correct


it usually growth quickly and spontaneus resolve

24. A hypertension patient with sudden onset pain behind left eye, with ptosis, palsy signs of VII, V, VI,
(paralysis of palate, eye movement paralyss), Dx?
a. aneurysm of posterior communication william ring (located in the midbrain)
b. midbrain infarct (carries the 3rd and 4th cranial nerves)
c. brain stem infarctment

25. Indication for thrombolytic therapy ?


a. new LBBB (Left bundle branch block (LBBB), unlike right bundle branch block, is always an indication of heart disease, most
commonly coronary artery disease)
b. new RBBB
c. Q waves (is an indicator of myocardial infarction)
d. T inversion (appears after infarction when the ST segment returns to normal but is also the result of other causes and could be
found in normal people)
e. ST depression (usually seen in ischemia rather than infarction)

26. a men presenting with chest pain mimic ischimic chest pain with prolonged period, which is correct?/
a. heparin and iv nitrate are not indicated (this is not related to the heart)
b. thrombolytic Tx can not given if ECG is normal

27. A young man with cough, Mantoux +, CXR showed inacticve TB lesion, Mx?
a. triple treatment for 6 months and repeat CXR
b. isoniazid tx fro 3 mons and repeat CXR
c. 6 sputum and await
d. repeat CXR in 3 months

28. a lady with diarrhoea, anaemia, fatigue, MCV 110


(Gastrointestinal causes of Macrocytosis include :stomach removal surgery, celiac disease [sprue], Crohn's disease)
a. pernicious anaemia
b. gluten-sensitive enterotomy
c. crohn’s disease

29. Blood gas ph 7.45 (N 7.35-7.44), pco2 27, po2 65, hCO3 23,
a. Respiratory alkalosis with gas change impairment
b. Respiratory alkalosis without gas change impairment

30. COPD which is the risk of right heart failure


a. FEV <1 L
b. Diffusing Capacity <50%
c. Reduced Po2 (HypoxiaPolycythemia + Airway remodelling Pulmonary hypertension RHF)
d. Increased Pco2

31. COPD patient was brought in by ambulance. Was given 10 L O2 on the way. ABG Pco2 65 Po2 60 and
Hco3 23.
a. reduce O2 and redo the ABG 30 mins later
32.Infective Endocarditis asssociated with
a. Vasculitis is the presentation of endocarditis (prolonged fever is the presentation)
b. staphal is the common cause (Streptococcus viridans is the most common)
c. it is the direct from rheumatic fever

33.The patient presents in the emergency department with BP-90/50


Pulse rate 98 min. CVP is 0.5 water. What is the possible cause? (Normal CVP is 2-6 mm Hg.)
1.Cardiac arrest
2.Hypovoluemic shock
3.Bowel obstruction
4.Pancreatitis

34.The patient presents with the severe pain which is radiating to the back, BP-180/95. What is the feature
you are going to find on the CXR:
1.Widened mediastinum ( A sign of aortic aneurysm)

35.The abattoir worker is presented with jaundice, fever, and malaise.


What is the possible DS: (Q fever could also occure in abattoir workers)
1.Leptosperosis (caused by exposure to the bacteria, which can be found in fresh water contaminated by animal urine, therefore it
occure in farmers, ranchers, abattoir workers or fresh water swimmers. But unlike brucellosis it is presented with Sever jaundice)
2.Brucellosis (Transmission of the disease to humans occurs by contact with infected meat)
3.Malaria
4.Dengue fever (Dengue fever is a viral illness transmitted by mosquitoes.)

36.Chronic diarrhoea for 3 months could be caused by except:


1.Campylobacter Jejuni(There is an incubation period of 2-4 days before symptoms occur. Symptoms generally last one week)
2.Crohn’s disease
3.Ulcerative colitis
4.Giardiasis
5. Laxative abuse

37.Most common cause perfuse bleeding in elderly:


1.Diverticulosis (sudden and painless profuse rectal bleeding)
2.Cancer of the colon (Occult blood)
3.Crohn disease (bloody diarrhea is not common)
4.Ulcerative colitis (occult blood)
5. ischemic colitis (Bright redblood in the stool associated with sever abdominal pain)

38 The patients presents with oedema of the face .Mx: treatment for hypertansion,gaaut and alopurinol
,indometacin.The most possible cause of his oedema:
1.Indometcin intoxication(stop the treatment) (angio-edema)
2.Renal failure (there are no urinary signs)

39.IgA nephropathy

40. a lady with oligio and creptitaion, poor prognosis is?


a. Haematuria is the poor prognosis

41. caute and chronic renal failure?


a. ultrasound

42. increase transferrin seen in:


a. haemochromatosis
43. A patient with COPD , given O2 10L/min, become unresponsive, which of F is possible blood gas
a. ph7.15, Po2 65, pco2 100

44. patient with heart failure and gout, treated with thiazide diuetics, ACE -or, alloperidol and
indomethocine. Test : K7, Na 1.25
a. Na low indicate Na depletion

45. Na of 122 indicates?


a. low plasma osmolority.

46. a long list of blood film


a. multiple myoloma

47. Indication of speletomy


a. chronic ITP

48. Asthma, all except


a. total lung capacity reduced.

49. indication for posterior column lesion?


a. Romberg’s test positive

50. Hypothyrodism, except?


a. small hand muscle atrophy

51. opening snap indicate?


a. mobility

52. What is not characteristic of Autosome recessive


a. 50/50 chance of inheriance

53. DM with autoneuropathy all except


a. urine incontinence
b. diarrhoea
c. bradycardia

54. Most reliable Dx for Giardia


a. stool exam of cyst (repeated stool examination maybe negative)
b. duodenal fluid

55. Coeleic Dis Dx to confirm?


a. Duodenal biopsy
b. Antibody-gliadin
c. Antibody-endomyial

56. infectious mononucleosis, all correct except,


a. is usually associated with hepatitis (Elderly patients with EBV present clinically as having anicteric viral hepatitis.)
b. Diagnosed by antigen detection (monospot test: detects the presence of heterophile antibodies)

57. angioplasty and stent comparing, which is correct?


(When angioplasty is performed alone, blockages can recur, although most of these arteries can be opened again successfully. This can
also occur when a stent is placed in the artery at the time of the angioplasty.)
a. reduce restenosis but same complications as angioplasty
b. more complications than angioplasty alone
c. no difference

You might also like